Вы находитесь на странице: 1из 90

Practice Questions 1

A1 C1 3
1. A forty-eight year-old female sustained a fracture to her left shoulder. Treatment
is proceeding well except that with left shoulder flexion you notice the scapula
protract and elevate early and it continues to move excessively. Physical therapy
intervention should emphasize:
A. glenohumeral mobilization and strengthening of scapular stabilizers to regain
normal scapulohumeral movement.
B. glenohumeral mobilization, and strengthening of the rotator cuff muscles to
regain muscle balance.
C. scapulothoracic mobilization and strengthening of the pectoralis major and
minor muscles to regain normal scapulohumeral rhythm.
D. stretching of scapular stabilizers and strengthening of the pectoralis major and
minor muscles to regain muscle balance.

A2 C8 1
2. A 59 year-old ex-machinist demonstrates significant age-related hearing loss,
presbycusis. When trying to communicate with this patient you would NOT
suspect:
A. bilateral hearing loss, at all frequencies since he has had this problem for a
number of years.
B. decreased language comprehension.
C. poor auditory discrimination.
D. unilateral hearing loss.

A3 C3 3
3. You are supervising the exercise of cardiac rehabilitation outpatient class on a
very hot day, with temperatures expected to be above 90 degrees F. Your class is
scheduled for 2 p.m. and the facility is not air conditioned. The strategy that is
unacceptable is to:
A. change the time of the exercise class to early morning or evening.
B. decrease the exercise intensity by slowing the pace of exercise.
C. increase the warm-up and cool-down periods to equal the total aerobic interval
in time.
D. make the exercise intermittent by adding rest cycles.

A4 C6 1
4. A patient has a transtibial amputation and has recently been fitted with a PTB
socket. During your initial prosthetic checkout, you instruct the patient to walk
several times in the parallel bars. You then have him sit down and take the
prosthesis off. You inspect the skin. You would expect no redness in the area of
the:
A. anterior tibia, tibial crest, and fibular head.
B. patellar tendon and tibial tuberosity.

’05 Sullivan A
2 Practice Questions

C. medial tibial and fibular plateaus.


D. distal end of the residual limb.

A5 C4 2
5. A patient who is terminally ill with cancer is in tears, unable to cope with the
changes in her life and with her current hospitalization. You have a referral for
gait training so she can be discharged to home under hospice care. The BEST
approach for you to take is:
A. ask the patient questions so you can gain a detailed history.
B. encourage denial so she can cope better with her life’s challenges.
C. ignore her tears and focus on her therapy but in a compassionate manner.
D. take time now to allow the patient to express her fears and frustrations.

A6 C5 3
6. Your examination reveals muscle spasms of the deep hip rotators, which are
compressing the sciatic nerve and producing pain in the posterior hip region. The
MOST effective setting of ultrasound in this case is:
A. 1 MHz continuous at 1.0 W/cm2.
B. 1 MHz pulsed at 1.0 W/cm2.
C. 3 MHz continuous at 1.0 W/cm2.
D. 3 MHz pulsed at 1.0 W/cm2.

’05 Sullivan A
Practice Questions 3

A7 C2 2
7. A 17 year-old patient is recovering from a complete spinal cord injury, at the
level of L2. The expected outcome in this case would MOST likely include:
A. a spastic or reflex bladder.
B. greater loss of arm function than leg function with early loss of pain &
temperature sensation.
C. loss of motor function, pain & temperature sensation below the level of the
lesion with light touch, proprioception and position sense preserved.
D. some recovery of function since damage is to peripheral nerve roots.

A8 C7 3
8. A physical therapist is gait training a patient with left hemiplegia. The patient’s
new AFO arrives, but the therapist is overwhelmed with too many patients and
asks the physical therapy student to take over. This is the student’s first affiliation
(second day) and she has never performed an orthotic checkout for a patient with
an AFO. The supervising therapist will be in the same vicinity treating other
patients. This task should:
A. be considered an advanced task, but allowable for the student to perform as a
good learning experience.
B. be designated as a more advanced task and more appropriately delegated to
another physical therapist.
C. be designated as a routine task and appropriate for the student who could call
out to the supervisor if problems arose.
D. not be completed now and the patient sent back to his room.

A9 C1 2
9. A patient has been diagnosed with acute synovitis of the temporomandibular
joint. Early intervention should focus on:
A. application of an intraoral appliance and phonophoresis.
B. instruction to eat a soft food diet and phonophoresis.
C. joint mobilization and postural awareness.
D. temporalis stretching and joint mobilization.

A10 C1 3
10. During a cervical spine examination you observe restricted left rotation of the C7-
T1 spinal level. After stabilizing the thoracic spine, your hand placement for
mobilization to improve left rotation should be at the:
A. posterior left C6 articular pillar.
B. posterior left C7 articular pillar.
C. posterior right C7 articular pillar.
D. T1 spinous process.

A11 C8 1

‘05 Sullivan A
4 Practice Questions

11. An appropriate fine motor behavior that should be established by 9 months of age
would be the ability to:
A. build a tower of 4 blocks.
B. hold a cup by the handle while drinking.
C. pick up a raisin with a fine pincer grasp.
D. transfer objects from one hand to another.

A12 C2 1
12. A patient with a 10 year history of Parkinson’s disease has been taking L-dopa for
the last 5 years. He presents in your clinic with deteriorating function. He is not
longer able to transfer or walk independently. During the oral interview, you
observe facial grimacing with twitching of the lips and tongue protrusion. He
appears restless, with constant dancing, athetoid-like movements of his legs. Your
BEST course of action is to:
A. complete your evaluation focusing specifically on his main problems of
rigidity and bradykinesia.
B. document your observations and refer him back to his physician for evaluation
of possible dopamine toxicity.
C. talk to his wife to see if he is taking any drugs with hallucinogenic effects.
D. use isokinetic dynamometry to assess his inability to generate torque output
during fast movements.

’05 Sullivan A
Practice Questions 5

A13 C7 3
13. A patient who is to undergo surgery for a chronic shoulder dislocation asks you to
explain the rehabilitation following a surgical reconstructive procedure he is
scheduled to undergo. Your BEST response is to:
A. explain how patients typically respond to the surgery and outline the
progression of exercises.
B. explain in detail about the surgical procedure.
C. refer the patient to a physical therapy clinical specialist who is an expert of
shoulder reconstructive rehabilitation.
D. tell the patient to ask his surgeon for this information.

A14 C3 2
14. Which of the following is NOT an appropriate reason to terminate a maximum
exercise tolerance test for a patient with pulmonary dysfunction?
A. ECG monitoring reveals diagnostic ischemia.
B. PaO2 decreases 20 mmHg.
C. patient reaches age-predicted maximal heart rate.
D. patient states he is maximally short of breath.

A15 C6 3
15. A patient with a complete T10 paraplegia is receiving his initial ambulation
training. He has received bilateral Craig-Scott knee-ankle-foot orthoses and is
being trained with axillary crutches. Since a reciprocal gait pattern is problematic
for him, the BEST initial gait pattern to teach him is:
A. four-point.
B. swing-through.
C. swing-to.
D. two-point.

A16 C4 2
16. After mastectomy, a patient cannot accept the loss of her breast. She reports being
weepy all the time with loss of sleep. She is constantly tired and has no energy to
do anything. The BEST action you can take is:
A. observe her closely for possible suicide.
B. request her primary physician to refer her for psychological evaluation.
C. tell her she’s over-reacting, she has to get on with her therapy.
D. tell the nurse case manager to monitor her behavior.

A17 C5 3
17. A 23 year-old college volleyball player complains of moderate pain resulting
from a left hamstring strain four weeks ago. The focal point of pain and tightness
is noted where a hematoma developed initially. The specific massage technique
that would be MOST beneficial in this case is:

‘05 Sullivan A
6 Practice Questions

A. friction.
B. kneading.
C. stroking.
D. tapotement.

A18 C8 2
18. A six year-old boy born with myelomeningocele at the L2 level is referred for
physical therapy treatment at home. In determining the physical therapy plan of
care, it would NOT be appropriate to emphasize:
A. gait training with a reciprocating gait orthosis.
B. transfer training from floor to wheelchair.
C. upper extremity strengthening with weights.
D. vigorous range of motion of the lower extremities.

A19 C1 1
19. During your examination of a patient who complains of back pain you found pain
with end range AROM into hip flexion, abduction and external/lateral rotation.
The structure most likely causing the patient’s pain is the:
A. hamstring muscle.
B. hip joint.
C. piriformis muscle.
D. SI joint.

A20 C7 2
20. A therapist wants to compare frequencies of carpal tunnel syndrome occurring in
different groups of individuals: assembly line workers and computer
programmers. The MOST appropriate statistical tool to use for analysis of the
data is:
A. chi square test.
B. normal distribution curve.
C. simple one-way ANOVA.
D. t test.

A21 C7 1
21. In a research study in which there is a skewed distribution with extreme scores on
a balance measure that deviate from the performance of the total group, the
MOST accurate representation of central tendency is:
A. mean.
B. median.
C. mode.
D. standard deviation

A22 C2 1

’05 Sullivan A
Practice Questions 7

22. A 54 year-old factory worker injured his right arm in a factory press with damage
to the ulnar nerve at the elbow. A diagnostic EMG was performed with evidence
of spontaneous fibrillation potentials. In this case, the physical therapy plan of
care should consider that:
A. axonotmesis is occurring.
B. denervation atrophy has occurred.
C. reinnervation is complete.
D. reinnervation is in process.

A23 C6 3
23. A 67 year-old patient is recovering from a left CVA resulting in severe right
hemiplegia. Additionally, he has a large diabetic ulcer on his left foot with pitting
edema, requiring elevation of that extremity. The MOST appropriate wheelchair
prescription for this patient would be a:
A. hemiplegic chair.
B. lightweight active duty wheelchair.
C. one-arm drive chair.
D. powered wheelchair with joystick.

A24 C3 1
24. A 14 year-old boy with advanced Duchenne muscular dystrophy is administered a
pulmonary function test. The value that is UNLIKELY to show any deviation
from normal is:
A. FEV1.
B. functional residual capacity.
C. total lung capacity.
D. vital capacity.

A25 C6 3
25. A 29 year-old woman fractured her right midtibia in a skiing accident three
months ago. After cast removal, a severe foot drop was noted. The patient desires
to try electrical stimulation orthotic substitution. You would set up the functional
electrical stimulation to contract the appropriate muscles during:
A. foot flat.
B. push off.
C. swing phase.
D. toe off.

A26 C4 2
26. You have a small area of dermatitis on the back of your hand with moderate
exudate. You are scheduled to treat a patient with HIV for management of an
open wound. You should:

‘05 Sullivan A
8 Practice Questions

A. continue with treatment as scheduled but wash your hands thoroughly before
and after.
B. double glove and treat as scheduled.
C. refuse to treat that patient.
D. use sterile precautions with mask and gloves.

A27 C1 2
27. You are a home therapist treating a patient who underwent a total hip replacement
four weeks ago. You notice that the patient arches his lumbar spine in supine. He
states that it is uncomfortable and doesn’t remember having the problem before.
The patient is unable to maintain a comfortable supine position due to:
A. poor abdominal strength.
B. tight hamstrings muscles.
C. tight iliopsoas muscle.
D. tight piriformis muscle.

A28 C8 3
28. Your 102 year-old patient has been hospitalized for the past three days with an
undisclosed ailment. After running numerous tests and finding nothing to explain
her increasing weakness and fatigue, the physicians are being pressured to
discharge her tomorrow. She lives alone in a first floor apartment. You have
determined her ambulation endurance to be only up to 15 feet, not enough to
allow her to get from her bed to the bathroom (a distance of 20 feet). You
recommend:
A. a live-in nurse (24 hour coverage) until her condition improves.
B. a skilled nursing facility placement until her endurance increases.
C. environmental changes, a bedside commode, and referral for home health
services.
D. postponing her discharge until she can complete the needed 20 feet.

A29 C2 3
29. You have determined a patient with a right CVA that you are currently treating
has a profound deficit of homonymous hemianopsia. The BEST initial strategy to
assist the patient in compensating for this deficit is to:
A. make the patient aware of his deficit and teach him to turn his head to the
affected left side.
B. place items, eating utensils on his left side.
C. provide constant reminders, printed notes on his left side, telling him to look
to the left.
D. rearrange his room so while in his bed his left side is facing the doorway.

A30 C2 2

’05 Sullivan A
Practice Questions 9

30. A patient with multiple sclerosis is agitated, irritated, and tired during treatment.
Your BEST response as the P.T. is to:
A. begin pool therapy to promote relaxation.
B. ignore these behaviors and continue treating.
C. reduce the exercise intensity and provide relaxation strategies.
D. treat the patient in a cool environment.

A31 C7 3
31. A 72 year-old medically stable individual requires custodial care in the home. She
is severely disabled with rheumatoid arthritis and is in a great deal of pain. She
presents with significant deformities which limit her functional abilities and is
dependent in all basic activities of daily living. A recent exacerbation of her
disease has left her bed-bound for the past 2 weeks. Appropriate physical therapy
services would be covered by:
A. Medicaid.
B. Medicare.
C. Medigap policies.
D. only by HMO or private insurance policies.

A32 C3 2
32. A patient with a purulent venous insufficiency ulcer near the left medial
malleolus is seen at home by a physical therapist. The MOST important
goal/intervention for the therapist to try to achieve with this patient is:
A. daily changes of elastic wraps to reduce fluid buildup.
B. daily warm water baths to improve circulation.
C. increase ambulation endurance to hasten wound healing.
D. instruct in proper dressing changes and wound care.

‘05 Sullivan A
10 Practice Questions

A33 C6 1
33. Your patient is having difficulty bearing weight on the left leg. She is unable to
advance the tibia forward and abbreviates the end of the stance phase on the left
going directly into swing phase. The MOST likely cause of her problem is:
A. hip extensor weakness.
B. spasticity of the anterior tibialis muscle.
C. spasticity or contracture of the plantarflexors.
D. weakness or contracture of the dorsiflexors.

A34 C1 1
34. With a traction injury to the anterior division of the brachial plexus you would
expect to see weakness of the elbow flexors, wrist flexors and forearm pronators.
You would also expect to find additional weakness in:
A. forearm supination.
B. lateral rotation of the shoulder.
C. thumb abduction.
D. wrist extension.

A35 C8 3
35. A 95 year-old has recently been admitted to your skilled nursing facility
following a fall-related injury (fractured hip with open reduction, internal
fixation). Since she lived alone on the second floor, she was unable to return
home. She is extremely agitated over her placement here and demonstrates early
signs of dementia. She tells you “leave me alone, I just want to get out of here!”
An important approach to take while working with this client is to:
A. be calm and supportive, using only one or two level commands.
B. establish the rules: tell her exactly what she is to do while you are with her.
C. use gesture or sign language to communicate with her.
D. promise her anything as long as she gets up and walks for you.

A36 C8 1
36. A 72 year-old woman is being treated for depression following the death of her
husband. She is currently taking antidepressant medication (tricyclics) and has a
recent history of a fall. You suspect the precipitating cause of the fall can be
attributed to side effects of her medication resulting in:
A. cardiac arrhythmias
B. dyspnea.
C. hyperalertness.
D. postural hypotension.

A37 C3 1
37. An 82 year-old patient and his caregivers should understand the common side
effects of the medication that he is taking. He is continually in and out of

’05 Sullivan A
Practice Questions 11

congestive heart failure and has been taking digitalis (Digoxin) to improve his
heart function. You will know he and his caregivers understand the adverse side
effects of this medication if they tell you they will contact the patient’s physician
if he demonstrates:
A. confusion and memory loss.
B. involuntary movements and shaking.
C. slowed heart rate.
D. weakness and palpitations.

A38 C7 3
38. The Director of Physical Therapy from a large teaching hospital is asked to
develop an operating budget for the upcoming fiscal year. The item that would
NOT be included in an operating budget is:
A. a treadmill purchase.
B. equipment maintenance.
C. housekeeping supplies.
D. long distance telephone calls.

‘05 Sullivan A
12 Practice Questions

A39 C4 1
39. You are treating a patient with active hepatitis B infection. Transmission of the
disease is best minimized if you:
A. avoid direct contact with any part of the patient.
B. have the patient wear a gown and mask every time your are in the room.
C. have the patient wear gloves to prevent him from touching you.
D. wear gloves if there is direct contact with blood or body fluids.

A40 C5 3
40. A wrestler complains of pain (7/10) and limited range of motion of the right
shoulder as a result of chronic overuse. You elect to use procaine hydrochloride
iontophoresis as part of your intervention for this patient’s problems. To
administer this substance, it would be appropriate to use:
A. continuous biphasic current with the medication under the anode.
B. continuous monophasic current with the medication under the anode.
C. continuous monophasic current with the medication under the cathode.
D. interrupted monophasic current with the medication under the cathode.

A41 C1 2
41. A contraindication to initiating joint mobilization on a patient with chronic
pulmonary disease may include:
A. concurrent inhalation therapy.
B. functional chest wall immobility.
C. long term corticosteroid therapy.
D. reflex muscle guarding.

A42 C8 3
42. Long term care for institutionalized elderly who have reduced their financial
resources or qualify for low-income status is typically funded by:
A. Health Maintenance Organizations.
B. Medicaid.
C. Medicare.
D. Social Security Administration.

A43 C3 1
43. A 45 year-old computer programmer, with no significant past medical history,
presents to the emergency room with complaints of fever, shaking chills and a
worsening productive cough. He has chest pain over the posterior base of his left
thorax which is made worse on inspiration. An anterior-posterior X-ray shows an
infiltrate on the lower left thorax at the posterior base. This patient’s chest pain is
MOST likely caused by:
A. angina.
B. infected pleura.

’05 Sullivan A
Practice Questions 13

C. inflamed tracheobronchial tree.


D. trauma to the chest.

A44 C7 3
44. Equipment safety is essential in all physical therapy clinics. Regularly scheduling
equipment maintenance programs to ensure that all equipment is calibrated,
lubricated, and adjusted according to manufacturer’s guidelines is an important
element for patient and staff safety. All of the following procedures should be
followed to ensure safety EXCEPT:
A. conducting educational sessions for staff regarding the indications and
contraindications for all equipment.
B. documenting all preventive maintenance and keeping this information on file.
C. supervising new staff and students in the use of all newly purchased
equipment.
D. training all staff to do simple repairs on all electrical equipment if a
breakdown should occur.

A45 C4 1
45. A 92 year-old woman presents with hot, red, and edematous skin over the shins of
both lower extremities. She also has a mild fever. The MOST likely cause of her
symptoms is:
A. cellulitis.
B. dermatitis.
C. herpes simplex infection.
D. scleroderma.

A46 C5 2
46. A 10 year-old presents with pain and limited ROM following surgical repair of
the medial collateral ligament and anterior cruciate ligaments. The modality that
would be CONTRAINDICATED in this case is:
A. interferential current.
B. shortwave diathermy.
C. transcutaneous electrical stimulation.
D. ultrasound.

A47 C6 3
47. Hypertrophy is the muscular response to strength training. This can be expected
to occur following at least:
A. 1-2 weeks of training.
B. 2-3 weeks of training.
C. 3-4 weeks of training.
D. 6-8 weeks of training.

A48 C1 3
‘05 Sullivan A
14 Practice Questions

48. A diagnosis of bicipital tendinitis has been made following an evaluation of a


patient with shoulder pain. The BEST shoulder position to expose the tendon of
the long head of the biceps for application of phonophoresis would be:
A. abduction.
B. external/lateral rotation and extension.
C. horizontal adduction.
D. internal/medial rotation and abduction.

A49 C7 3
49. A patient is unable to bring her foot up on the next step during a training session
on stair climbing. Your BEST course of action to promote active learning is to
have the patient:
A. balance on the stairs while you passively bring the foot up.
B. practice marching in place in the parallel bars.
C. practice standing-up from half-kneeling.
D. step up onto a low step while in the parallel bars.

A50 C2 3
50. A 76 year-old patient exhibits impaired balance. A diagnostic work-up has failed
to reveal any specific etiology. An initial intervention for this patient would NOT
include:
A. limits of stability re-education including postural sway training.
B. practice in maintenance of a wide base of support during gait and turns.
C. sit-to-stand and stand-to-sit activity training.
D. tandem walking and single limb stance.

A51 C6 3
51. A 38 year-old patient with spinal cord injury is being discharged home after a 2
month course of rehabilitation. In preparation for discharge, you and your team
visit the home and find he has 3 standard height steps going into his home. A
ramp will have to be constructed for his wheelchair. The recommended length of
his ramp should be:
A. 120 inches (10 feet).
B. 192 inches (16 feet).
C. 252 inches (21 feet).
D. 60 inches (5 feet).

A52 C3 1
52. Following pneumonectomy, an expected change in a patient’s status would NOT
include:
A. asymmetrical breathing.
B. decreased breath sounds.
C. deviated trachea.

’05 Sullivan A
Practice Questions 15

D. increased tidal volume.

‘05 Sullivan A
16 Practice Questions

A53 C3 1
53. An older person with diagnosis of congestive heart failure (CHF) should
recognize the symptoms of exertional intolerance. You will know this client is
properly informed if she can tell you these symptoms are:
A. dizziness, visual blurring especially with turns and quick movements.
B. overwhelming weakness with difficulty in standing up and walking.
C. severe, uncomfortable chest pain with shortness of breath.
D. shortness of breath at rest and with limited activity, and sudden weight gain

A54 C6 3
54. You are instructing a physical therapy student in proper positioning to prevent the
typical contractures in the patient with a transtibial amputation. You stress:
A. maximize out-of-bed time with sitting in a chair.
B. position in prone lying and sitting with full knee extension.
C. position in prone-lying with slight knee flexion.
D. position in supine with a small pillow under the knee.

A55 C4 2
55. You receive a referral to ambulate a patient who is insulin dependent. In a review
of her medical record, you notice her blood glucose level for that day is 310
mg/dL. Your BEST course of action is to:
A. ambulate the patient as planned but monitor closely.
B. postpone therapy and coordinate with the nurse regarding insulin management
and exercise.
C. refrain from ambulating the patient, reschedule for tomorrow.
D. talk to the nurse about seeing the patient later on that day.

A56 C8 3
56. A physical therapy plan of care for a newborn with Erb-Klumpke Palsy would
NOT include:
A. age appropriate movements of the upper extremity.
B. gentle ROM after immobilization.
C. partial immobilization of limb across abdomen.
D. splinting the shoulder in abduction and internal rotation.

A57 C1 3
57. A six month-old child was referred to physical therapy for right torticollis. The
MOST effective method to stretch the muscle is by positioning the head and neck
into:
A. extension, left side-bending, and right rotation.
B. extension, right side-bending, and left rotation.
C. flexion, left side-bending, and left rotation.
D. flexion, right side-bending, and left rotation.

’05 Sullivan A
Practice Questions 17

A58 C7 3
58. A student you are supervising is on final affiliation following completion of
academic training. He is overheard discussing a patient’s history in the elevator.
When you later point this out to the student, he tells you he was unaware of any
hospital policy regarding confidentiality. Your BEST analysis of this situation is
that:
A. compliance was not a realistic expectation since he just arrived at this facility.
B. he should be expected to value patient confidentiality.
C. now that he is aware of confidentiality restrictions he should do better the next
time.
D. since this is not strictly part of the professional code of ethics you should not
reasonably expect him to demonstrate adherence to this concept.

A59 C7 2
59. A therapist is performing clinical research in which a specific myofascial
technique is applied to a patient with chronic back pain. She is using a single case
experimental design with an A-B-A-B format. Her research hypothesis states that
pain rating scores will decrease with the treatment intervention. Acceptance of
this hypothesis would be indicated if:
A. B is equal to A.
B. B is greater than A, at the .05 level.
C. B is greater than A, at the 1.0 level.
D. B is less than A.

A60 C2 3
60. Isokinetic training can best be used in the rehabilitation of patients with stroke
during the late stages of recovery to:
A. improve initiation of movement.
B. improve rate control at faster movement speeds.
C. improve rate control at slower movement speeds.
D. increase strength of synergy components.

A61 C6 3
61. A 72 year-old patient with a transfemoral amputation is unable to wrap his
residual limb. Your BEST course of action is to:
A. apply a temporary prosthesis immediately.
B. consult with the vascular surgeon about the application of an Unna’s paste
dressing.
C. redouble efforts to teach proper wrapping.
D. use a shrinker.

A62 C3 2

‘05 Sullivan A
18 Practice Questions

62. A 75 year-old patient with peripheral vascular disease has been referred for
conditioning exercise. He demonstrates moderate claudication pain in both legs
following a 12 minute walking test. The MOST appropriate exercise frequency
and duration for this patient is:
A. 2 times/week, BID 20 minutes/session.
B. 3 times/week, 30 minutes/session.
C. 3 times/week, 60 minutes/session.
D. 5 times/week, BID 10 minutes/session.

A63 C1 1
63. The radiographic view shown in the diagram that demonstrates the observed
spinal defect is: Twomey L, Taylor J (2000) Physical Therapy of the Low Back,
3rd ed. Philadelphia, Churchill Livingstone, Figure 7-1, page 204, with
permission.
A. frontal.
B. lateral.
C. oblique.
D. posterolateral.

A64 C4 2
64. A physical therapist is working with a patient with metastatic breast cancer who
has been told that she has only months to live. She is quite angry and disruptive
during therapy. What is the MOST appropriate intervention for this patient?
A. allow the patient to express her anger while refocusing her on effective coping
strategies.
B. forbid all expressions of anger as she is only hurting herself.
C. provide honest, accurate information about her illness and rehab plan of care.
D. provide opportunities for the patient to question her impending death but limit
all expressions of anger.

A65 C1 1
65. A forty year-old female cafeteria worker sustained a right-sided injury to her back
while playing golf. She was driving the ball when she felt an immediate sharp
pain in her right low back. She states that in the morning she is stiff and her pain
eases after taking a shower. Based on the above information, the structure MOST
likely involved is:
A. a disc.
B. a facet joint.
C. a nerve root.
D. the arterioles which supply circulation to the spinal cord.

’05 Sullivan A
Practice Questions 19

A66 C7 3
66. At 10 a.m. a physical therapist working on a spinal cord unit is treating a patient
with paraplegia at the T3 level. The therapist smells alcohol on his breath and the
patient is having difficulty accomplishing a bed-to-chair transfer that was
previously done without assistance. In this case the physical therapist should:
A. confront the patient and ask if he has been drinking.
B. document and report suspicions of alcoholism to the rehabilitation team at the
weekly meeting.
C. document the findings and immediately inform the patient’s physician about
the situation.
D. immediately inform the nurse in charge.

A67 C2 3
67. A patient with right hemiparesis has difficulty clearing the affected foot during
the swing phase of gait. An appropriate physical therapy intervention for the right
lower extremity might include:
A. bridging.
B. forward step-ups in standing using graduated height steps.
C. pushing backward while sitting on a rolling stool.
D. sitting on a Swiss ball, alternating lateral side steps and back to neutral.

A68 C6 2
68. A 19 year-old patient has a complete spinal cord injury at the level of L1. His
primary goal is to walk again. You decide it would be MOST appropriate to
recommend that this patient use:
A. a reciprocating gait orthosis and walker.
B. a wheelchair, because ambulation is unrealistic.
C. bilateral AFOs and Lofstrand crutches.
D. bilateral KAFOs with thoracolumbosacral extension control.

A69 C3 2
69. A patient is four weeks post myocardial infarction. Resistive exercises using
weights are:
A. always contraindicated during acute and post-acute phases of cardiac
rehabilitation due to expected elevations of BP.
B. appropriate if intensities are kept below 40% maximal voluntary contraction.
C. appropriate if intensities are kept below 85% maximal voluntary contraction.
D. safe during all phases of rehabilitation if judicious monitoring of HR is used.

A70 C5 3
70. A patient diagnosed with cervical radiculitis has been referred to you for
mechanical traction. You are applying the traction using the cervical halter for 5

‘05 Sullivan A
20 Practice Questions

minutes at 20° neck flexion using 10 pounds. The patient complains of pain in the
area of the TMJ. You should:
A. change the angle of pull.
B. decrease the traction poundage.
C. decrease the treatment time.
D. discontinue the treatment.

A71 C4 3
71. On the first day following a cesarean delivery, the physical therapist’s initial
intervention would consist of teaching the new mother:
A. ankle exercises to prevent thrombophlebitis.
B. assisted ambulation.
C. assisted breathing and coughing and pelvic floor exercises.
D. partial sit-ups and pelvic floor exercises.

A72 C1 2
72. It is important to note the status of the pars interarticularis on the X-ray report. A
problem with this part of the vertebra could possibly lead to:
A. spondylolisthesis with discal herniation.
B. spondylolisthesis with possible anterior slippage of the vertebral body.
C. spondylolysis resulting in early nerve root compression.
D. spondylolysis with early degeneration of the vertebra.

’05 Sullivan A
Practice Questions 21

A73 C8 3
73. The most appropriate school physical therapy intervention to use during class for
a child with decreased sitting balance, but normal tone would be to:
A. adapt a desk and wheelchair to provide adequate sitting balance.
B. use a prone-stander.
C. use a sidelyer.
D. use a therapeutic ball to promote sitting balance.

A74 C2 2
74. A patient with Parkinson’s disease demonstrates a highly stereotyped gait pattern
characterized by impoverished movement. The intervention that would be
LEAST appropriate to use with this patient is:
A. a rolling walker to compensate for impaired balance.
B. postural exercises to promote trunk and head extension in sitting and standing.
C. range of motion exercises to reduce hip and knee flexion contractures.
D. sidelying, upper and lower trunk rotation segmental patterns using the
technique of rhythmic initiation.

A75 C2 1
75. Your patient is 72 and recovering from a right CVA. She tells you she is thirsty
and asks you for a can of soda. When you give her the can and instruct her to
open it, she is unable to complete the task. Later after the treatment session when
she is alone you observe her drinking from the can. You suspect she may have a
primary deficit in:
A. anosognosia.
B. ideational apraxia.
C. ideomotor apraxia.
D. unilateral neglect.

A76 C7 3
76. Peer review is an important professional activity. Recently, physical therapists
have been the focus of vigorous peer review due to increasing financial pressure
imposed by third party payers. An inappropriate use of peer review is to
determine whether care:
A. should be paid for by a third party payer.
B. was appropriate and required the skill of a physical therapist.
C. was cost effective.
D. was provided by the appropriate personnel.

A77 C3 1
77. A 62 year-old patient has chronic obstructive pulmonary disease. The MOST
likely pulmonary test result would be:
A. decreased functional residual capacity.

‘05 Sullivan A
22 Practice Questions

B. decreased residual volume.


C. increased total lung capacity.
D. increased vital capacity.

A78 C6 1
78. A patient with a transfemoral amputation is being fitted with a quadrilateral
socket. Areas of pressure tolerance would be expected over the:
A. distolateral end of femur and ischial seat.
B. gluteals and adductor magnus.
C. ischial tuberosity, gluteals, and lateral sides of residual limb.
D. perineal area and medial side of femur.

A79 C4 2
79. A patient recently diagnosed with fibromyalgia and chronic fatigue immune
system dysfunction demonstrates a loss of interest in all activities and outlets. She
is not eating well and is having problems sleeping. Recently she has talked about
suicide as her only hope. Your BEST course of action is to:
A. call her physician, saying that you cannot do anything for this patient until her
psychological outlook is better.
B. discuss her need for medications with her husband.
C. immediately contact her primary physician.
D. present a positive attitude and tell her she will feel better soon.

A80 C6 3
80. A 77 year-old patient has been confined to bed for a period of 2 months and now
demonstrates limited ROM in both lower extremities. Range in hip flexion is 5º to
115º and knee flexion is 10º to 120º. The MOST appropriate intervention to
improve flexibility and ready this patient for standing is:
A. dynamic lower extremity splints, applied for 2 hours daily.
B. hold-relax techniques followed by passive ROM 3 times a week.
C. manual passive stretching, 5 repetitions each joint, 2 times a day.
D. mechanical stretching using traction and 5 lb. weights, 2 hours, twice daily.

A81 C1 2
81. A seventy year-old male retired carpenter who has had long term lumbar pain
with a previous diagnosis of degenerative joint disease (DJD) of his lumbar facet
joints. He complains of numbness, paresthesias and weakness of his bilateral
lower extremities which increase with extended positions or walking greater than
100 feet. His pain persists for hours after assuming a resting position. He reports
he can ride his stationary bike for 30 minutes without any problems. Primary
physical therapy intervention should include:
A. abdominal and back extension strengthening as the result of spondylolysis.
B. increasing cardiovascular endurance as the result of degenerative arthritis.

’05 Sullivan A
Practice Questions 23

C. stretching and limiting extended spinal positions as the result of spinal


stenosis.
D. traction and limitation of weight bearing positions as the result of discal
dysfunction.

A82 C7 3
82. A physical therapist is instructing a kindergarten teacher in a behavior
management program for a child with developmental disabilities who has been
mainstreamed into the regular classroom. The therapist requests that the teacher
encourage the child to maintain a head retracted sitting position in the class. The
strategy that would be MOST helpful in this situation is to:
A. have the teacher encourage the classmates to tell the child to sit up in the chair.
B. have the teacher give a smile sticker when the child sits with head retracted for
five minutes.
C. have the teacher issue a verbal reprimand when the child slumps in the chair.
D. train the teacher in manual handling techniques to assist the child in head
retraction.

A83 C7 3
83. You have recently attended a professional conference on myofascial release. You
wish to share this information with your colleagues during an inservice session.
You BEST initial activity is to:
A. ask your colleagues about their current level of knowledge using a brief
questionnaire.
B. determine the best sequence for the learning units within your presentation.
C. provide a comprehensive packet of handouts when you begin the first
inservice session.
D. select a suitable time and place for your lecture.

A84 C2 2
84. Following an episode of adhesive capsulitis of the right shoulder, a 52 year-old
with a history of left CVA now exhibits reflex sympathetic dystrophy affecting
the right upper extremity. The intervention that should be AVOIDED in this case
is:
A. graduated active exercises.
B. massage to reduce edema.
C. passive manipulation to the shoulder.
D. passive ROM exercises.

A85 C6 3
85. To prepare a patient with an incomplete T12 paraplegia for ambulation with
crutches, the upper quadrant muscles that would be MOST important to
strengthen include the:

‘05 Sullivan A
24 Practice Questions

A. deltoid, coracobrachialis, and brachialis.


B. lower trapezius, latissimus dorsi, and triceps.
C. middle trapezius, latissimus, dorsi and triceps.
D. upper trapezius, rhomboids, and levator scapulae.

’05 Sullivan A
Practice Questions 25

A86 C3 2
86. A 72 year-old is recovering at home from a myocardial infarction and
percutaneous transluminal coronary angioplasty. As his home care physical
therapist, you decide to use pulse oximetry to monitor his responses to exercise.
An acceptable oxygen saturation rate (SaO2) to maintain throughout the exercise
period is:
A. 50%.
B. 82%.
C. 85%.
D. 92%.

A87 C5 2
87. A 42 year-old homemaker presents with acute lateral epicondylitis following
participation in a local tennis tournament. Pain is 9/10. The MOST appropriate
initial intervention for this problem is:
A. cold intermittent compression three times per week.
B. cold whirlpool daily until the pain subsides.
C. fluidotherapy three times per week.
D. ice massage B.I.D. until the pain subsides.

A88 C1 1
88. A patient was referred to physical therapy complaining of loss of cervical AROM.
His X-rays showed DJD at the uncinate processes in the cervical spine. The
motion that would be MOST restricted would be:
A. extension.
B. flexion.
C. rotation.
D. side-bending.

A89 C1 2
89. A 55 year-old male electrician is unable to pull wire overhead due to a painless
inability to reach past 80 degrees of right shoulder abduction. He has had
numerous previous episodes of right shoulder pain over the last ten years which
were diagnosed as shoulder tendinitis. Early subacute physical therapy
intervention should focus on:
A. active assistive pulley exercises.
B. gentle grade III translatory glenohumeral mobilizations.
C. modalities to reduce pain and inflammation.
D. resistance exercises for the affected supraspinatus muscle.

A90 C7 2
90. Two therapists are asked to perform a test on the same group of patients using the
Functional Independence Measure (FIM). The results of both sets of

‘05 Sullivan A
26 Practice Questions

measurements reveal differences in therapists’ scores but not in the repeat


measurements. This is indicative of a problem in:
A. concurrent validity.
B. construct validity.
C. interrater reliability.
D. intrarater reliability.

A91 C2 3
91. Your patient is a 16 year-old recovering from a complete spinal cord injury with
C5 quadriplegia. You are performing PROM exercises on the mat when he
complains of a sudden pounding headache and double vision. You notice he is
sweating excessively, and when you take his BP it is 240/95. Your BEST course
of action is to:
A. immediately contact the patient’s physician.
B. lie the patient down immediately, elevate his legs, then call for a nurse.
C. place him in sitting position and continue to monitor BP.
D. sit the patient up, check/empty catheter, and then call for emergency medical
assistance.

’05 Sullivan A
Practice Questions 27

A92 C6 3
92. A 46 year-old patient presents with a flatfoot deformity with abduction of the
forefoot in relation to the weight bearing line. The forefoot is inverted to the varus
position when inspected from the frontal plane. Corrections for this foot
deformity would NOT include a:
A. metatarsal bar.
B. scaphoid pad.
C. Thomas heel.
D. UCBL insert.

A93 C4 3
93. A 10-year old with full thickness burns to both arms is developing hypertrophic
scars The BEST intervention to manage these scars is:
A. custom made pressure garments.
B. occlusive dressings.
C. primary excision followed by autografts.
D. surgical resection (Z-plasty).

A94 C3 1
94. A patient with COPD has developed respiratory acidosis. You instruct the
physical therapy student who is participating in her care to monitor the patient
closely for:
A. dizziness or syncope.
B. dyspnea, anxiety, or disorientation.
C. muscle twitching or tetany.
D. tingling or numbness.

A95 C1 2
95. A 34 year-old female in her second trimester of pregnancy was sent to physical
therapy with complaints of tingling and loss of strength in both of her hands. Her
symptoms are exacerbated if she is required to use her keyboard at work for
longer than 20 minutes. The MOST appropriate physical therapy intervention
would include:
A. dexamethasone phonophoresis to the carpal tunnel.
B. hydrocortisone iontophoresis to the volar surfaces of both wrists.
C. ice packs to the carpal tunnel.
D. placing the wrists in resting splints.

A96 C8 3
96. A child you are working with in the elementary school system has moderate to
severe extensor spasticity and limited head control. The MOST appropriate
positioning device would be a:
A. prone stander with abduction wedge.

‘05 Sullivan A
28 Practice Questions

B. supine stander with abduction wedge.


C. wheelchair with a back wedge and head supports.
D. wheelchair with adductor pommel.

A97 C2 3
97. A patient with complete C6 quadriplegia should be instructed to initially transfer
with a sliding board using:
A. pectoral muscles to stabilize elbows, scapular depressors to lift trunk.
B. serratus anterior to elevate trunk with shoulder extensors stabilizing.
C. shoulder extensors, external rotators, and anterior deltoid to position and lock
the elbow.
D. triceps, keeping the hands flexed to protect tenodesis grasp.

A98 C7 3
98. You have volunteered to teach a stroke education class on positioning techniques
for family members and caregivers. There will be 12 individuals attending this
class, ranging in age from 42 to 82. Your BEST choice of teaching methods is to
utilize:
A. demonstration, practice, and follow-up discussion.
B. lecture with some time for questions at the end of the 30 minute session.
C. multimedia (slides and overheads) to accompany your oral presentation.
D. one on one practice with adequate critiquing of performance.

’05 Sullivan A
Practice Questions 29

A99 C3 2
99. A patient with active tuberculosis is referred for physical therapy. Which of the
following is NOT an appropriate precaution?
A. have the patient wear a tight fitting mask while being treated in his room.
B. insure that the patient is in a private, negative pressurized room.
C. wash hands upon entering and leaving the patient’s room.
D. wear a tight fitting mask while treating the patient.

A100 C1 2
100. A twelve-year old female figure skater has been referred to your clinic for
treatment for patellar tendinitis. The examination reveals that she is unable to hop
on the affected lower extremity due to pain. You decide to refer her back to her
pediatrician suggesting that the patient receive an x-ray of her knee. The patient
returns for therapy with the x-ray shown in the figure. Your initial intervention
should focus on: Magee D (2002). Orthopedic Physical Assessment, 4th ed.
Philadelphia, W. B. Saunders, Figure 12-149, page 746, with permission.
A. aggressive plyometric exercises with focus on endurance training.
B. fitting patient with crutches for non weight-bearing ambulation and initiation
of phonophoresis at 3mHz continuous wave using hydrocortisone.
C. patient education regarding avoidance of squatting and jumping activities as
well as initiation of iontophoresis using dexamethasone.
D. patient education regarding avoiding falls onto her affected knee and open
chain knee extension exercises to improve quadriceps strength.

A101 C6 2
101. A 40 year-old male with a history of low back pain has been receiving physical
therapy for 12 weeks. The patient is employed as a loading dockworker. He
performs repetitive lifting and carrying of boxes weighing between 15 and 30
pounds. An appropriate engineering control to reduce the stresses of lifting and
carrying would be to:
A. issue the employee a back support belt.
B. provide a two-wheel handcart for use in moving the boxes.
C. require the worker to attend a class in using correct body mechanics while
performing the job.
D. use job rotation.

A102 C1 1
102. Common compensatory postures you would expect for a patient diagnosed with
fixed severe forefoot varus are:
A. excessive ankle dorsiflexion and medial rotation of the femur.
B. excessive midtarsal supination and lateral rotation of the tibia.
C. subtalar pronation and medial rotation of the tibia.
D. toeing-in and lateral rotation of the femur.

‘05 Sullivan A
30 Practice Questions

A103 C1 2
103. A patient has undergone surgery and subsequent immobilization to stabilize the
olecranon process. The patient now exhibits an elbow flexion contracture. In this
case, an absolute CONTRAINDICATION for joint mobilization would be:
A. empty end-feel.
B. firm end-feel.
C. soft end-feel.
D. springy end-feel.

A104 C6 3
104. You observe a physical therapist assistant ambulate a patient for the first time
after a left total hip replacement. The patient is using crutches and is practicing on
a level surface. The PTA should guard the patient by standing slightly:
A. behind and to the intact side, one hand on the gait belt.
B. behind and to the left side, one hand on the gait belt.
C. behind the patient with both hands on the gait belt.
D. in front of the patient, walking backward, with one hand on the gait belt and
one hand on the shoulder.

’05 Sullivan A
Practice Questions 31

A105 C5 1
105. A 13 year-old severed the median nerve three days ago when his hand went
through a glass window. To determine the motor function of the nerve you
perform a chronaxie test. At this time you would expect the chronaxie of the
nerve to be:
A. absent.
B. decreased.
C. increased.
D. unaffected.

A106 C1 3
106. A patient has lumbar spinal stenosis encroaching on the spinal cord. The physical
therapist should educate the patient to AVOID:
A. bicycling.
B. rowing.
C. swimming using a crawl stroke.
D. Tai Chi.

A107 C6 2
107. A patient with post-polio syndrome presents in your clinic with symptoms of
myalgia and increasing fatigue. He is wearing a KAFO which he has had for 10
years. When walking, you observe that he rises up over the sound limb to advance
the orthotic limb forward. Your BEST intervention is to provide:
A. a manual wheelchair with reclining back and elevating legrests.
B. a shoe lift on the orthotic side.
C. a shoe lift on the sound side.
D. an electric wheelchair with joystick.

A108 C5 2
108. Your patient has moderate spasticity of the biceps brachii on the left as a result of
a CVA. You choose to use electrical stimulation to temporarily decrease the
effects of hypertonicity in order to work on ADL activities. Your objective in
applying the current is to:
A. fatigue the ipsilateral biceps brachii.
B. stimulate the contralateral biceps brachii.
C. stimulate the contralateral triceps.
D. stimulate the ipsilateral triceps.

A109 C8 2
109. You are working with a four year-old child who has myelodysplasia at the L5
level. At this level the most appropriate orthosis to recommend for ambulation
would be a (an):
A. ankle-foot orthosis.

‘05 Sullivan A
32 Practice Questions

B. knee-ankle-foot orthosis.
C. parapodium.
D. reciprocating gait orthosis.

A110 C1 2
110. An eleven-year-old male was referred to physical therapy with complaints of
vague pain at his right hip and thigh which radiates to his knee. His AROM is
restricted in abduction, flexion, and internal rotation. A gluteus medius gait was
observed with ambulation for 100 feet. Appropriate PT intervention would
include:
A. closed-chain partial weight-bearing lower extremity exercises for slipped
capital femoral epiphysis.
B. hip joint mobilization to improve the restriction in motion as the result of
Legg-Calvé Perthe’s disease.
C. open-chain strengthening of his right hip abductors and internal rotators for
avascular necrosis of the hip.
D. orthoses to control lower extremity position as the result of femoral
anteversion.

’05 Sullivan A
Practice Questions 33

A111 C7 2
111. A researcher states that he expects that there will be no significant difference
between 20 and 30 year-olds after a 12 week exercise training program using
exercise heart rates and myocardial oxygen consumption as measures of
performance. The kind of hypothesis that is being used in this study is a (an):
A. directional hypothesis.
B. experimental hypothesis.
C. null hypothesis.
D. research hypothesis.

A112 C7 3
112. A physical therapist was treating a patient and the patient in the next bed was
uncomfortable and asked the therapist to move his leg. The therapist placed the
leg on 2 pillows as requested by the patient. Unknown to the therapist this patient
had a femoral artery graft 2 days previously. As a result the graft became
occluded and the patient was rushed to surgery for a replacement. The patient
claimed the therapist placed his leg too high on the pillows causing the occlusion
of the original graft and sued for malpractice. The hospital administrator decided:
A. it was the patient’s fault for requesting the position change and therefore
supported the action of the physical therapist.
B. that the physical therapist was functioning according to common protocols of
the institution and thus supported the actions of the therapist.
C. that the therapist was functioning outside the common protocols of the
hospital, and therefore did not support the actions of the physical therapist.
D. to counter-sue the patient because he was responsible for requesting the
position change.

A113 C2 3
113. A 26 year-old female has a 3 year history of multiple sclerosis. One of her
disabling symptoms is a persistent and severe diplopia which leaves her
frequently nauseated and immobile. An appropriate intervention strategy to assist
her in successfully participating in rehabilitation would be to:
A. give her a soft neck collar to limit head and neck movements.
B. give her special glasses which magnify images.
C. have her close her eyes and practice movements without visual guidance.
D. patch one eye.

A114 C2 3
114. The MOST appropriate positioning strategy for a patient recovering from acute
stroke who is in bed and demonstrates a flaccid upper extremity is:
A. sidelying on the affected side with the affected upper extremity flexed
overhead.

‘05 Sullivan A
34 Practice Questions

B. sidelying on the sound side with the affected upper extremity supported on a
pillow with the shoulder protracted and elbow extended.
C. supine with the affected hand positioned on stomach.
D. supine with the affected upper extremity positioned close to the side of the
trunk.

A115 C3 2
115. A patient presents with a large plantar ulcer that will be debrided in the whirlpool.
The foot is cold, pale, and painless. The condition that would most likely result in
this clinical presentation is:
A. acute arterial insufficiency.
B. chronic arterial insufficiency.
C. chronic venous insufficiency.
D. deep venous thrombosis.

A116 C5 3
116. A patient has been referred to you following a fracture of the femur six months
ago. The cast was removed, but the patient was unable to volitionally contract the
quadriceps. You decide to apply electrical stimulation to the quadriceps muscle.
Your choice of electrode placement and electrical stimulation duty cycle (on:off
ratio) would consist of:
A. large electrodes, closely spaced; 10:30.
B. large electrodes, widely spaced; 10:30.
C. small electrodes, closely spaced; 10:30.
D. small electrodes, widely spaced; 10:10.

A117 C4 2
117. While setting a patient up for cervical traction, you notice a purplish mole with
rough edges on the patient’s neck. You:
A. call the physician immediately and report your findings.
B. document the skin condition and keep a watchful eye on it.
C. tell the patient if it bleeds at all to report it to his physician.
D. treat the patient but cover the mole with a gauze pad.

A118 C2 1
118. A 15 year-old male suffered traumatic brain injury and multiple fractures
following a motor vehicle accident. He is recovering in the intensive care unit.
Your referral states PROM and positioning. On day 1 he is semi-alert and drifts in
and out while you are working with him. On day 2 you become concerned
because you observe signs suggestive of increasing intracranial pressure. You
promptly report these symptoms to his physician. The signs that would be cause
for immediate action in this case would be:

’05 Sullivan A
Practice Questions 35

A. decreasing consciousness with slowing of pulse and Cheyne-Stokes


respirations.
B. decreasing function of cranial nerves IV, VI, and VII.
C. developing irritability with increasing symptoms of photophobia,
disorientation and restlessness.
D. positive Kernig’s sign with developing nuchal rigidity.

A119 C2 1
119. You have received a referral for a 42 year-old patient who has a neurapraxia
involving the ulnar nerve secondary to an elbow fracture. Based on your
knowledge of this condition, you expect that:
A. regeneration is unlikely because surgical approximation of the nerve ends was
not performed.
B. nerve dysfunction will be rapidly reversed, generally in 2-3 weeks.
C. regeneration is likely after 2-21/2 years.
D. regeneration is likely in 6-8 months.

A120 C7 3
120. After three weeks of teaching a patient how to ambulate with bilateral crutches
and a touch down gait, you determine the most appropriate kind of feedback to
give to the patient is:
A. continuous feedback in which you provide ongoing verbal cuing during gait.
B. immediate feedback given after each practice trial.
C. intermittent feedback given at scheduled intervals, every other practice trial.
D. occasional feedback given when consistent errors appear.

A121 C8 2
121. You are a home health physical therapist. During one of your regularly scheduled
visits with a 72 year-old male patient, you find him to be confused with shortness
of breath and significant generalized weakness. Assessing these symptoms and
given his history of hypertension and hyperlipidemia, you suspect:
A. he forgot to take his hypertension medication.
B. he may be experiencing unstable angina.
C. he may be presenting with early signs of myocardial infarction.
D. his mental changes are indicative of early Alzheimer’s disease.

A122 C1 2
122. Physical therapy intervention for a sixty-five year-old male patient with a recent
diagnosis of supraspinatus tendinitis with possible impingement syndrome of the
right shoulder should emphasize:
A. joint mobilization, use of ice, and rotator cuff strengthening.
B. modalities to reduce inflammation, active assistive range of motion exercises
using pulleys, and postural realignment.

‘05 Sullivan A
36 Practice Questions

C. reducing stresses to abnormal tissues by placing the right upper extremity in a


sling, use of ice, and rotator cuff strengthening.
D. rest to reduce pain, iontophoresis, and strengthening of the rotator cuff
muscles.

’05 Sullivan A
Practice Questions 37

A123 C3 3
123. The recommended time duration for endotracheal suctioning is:
A. 1 to 5 seconds.
B. 10 to 15 seconds.
C. 15 to 20 seconds.
D. 5 to 10 seconds.

A124 C3 3
124. A 76 year-old frail older adult is confined to bed in a nursing facility. He has
developed a small superficial wound over the sacral area. Since only small
amounts of necrotic tissue are present, the physician has decided to use autolytic
wound debridement. This is BEST achieved with:
A. forceful irrigations.
B. hydrotherapy.
C. occlusive dressings.
D. wet-to-dry dressings.

A125 C5 3
125. Three weeks ago, you instructed your patient in applying conventional (high rate)
TENS to the low back to modulate a chronic pain condition. The patient now
states that the TENS unit is no longer effective in reducing the pain in spite of
increasing the intensity to maximum . You should now advise the patient to:
A. decrease the pulse duration.
B. increase the treatment frequency.
C. switch to low rate TENS.
D. switch to modulation mode TENS.

A126 C2 1
126. During initial standing a patient is pushing backward displacing the center-of-
mass at or near the posterior limits of stability. The most likely cause of this is
contraction of the:
A. gastrocnemius-soleus.
B. hamstrings.
C. hip extensors.
D. tibialis anterior/peroneals.

A127 C7 2
127. A patient has been screened using a new test for the presence of a gene (ALG-2)
linked to Alzheimer’s disease. His physician reports he lacks the gene and should
not be at increased risk to develop the disease. Some years later he develops
Alzheimer’s and a repeat test reveals the presence of the gene. The results of the
initial test can be interpreted as:
A. false negative.

‘05 Sullivan A
38 Practice Questions

B. false positive.
C. high degree of sensitivity.
D. high degree of specificity.

A128 C8 1
128. A 62 year-old lives at home with his wife and adult daughter. He has recently
been diagnosed with multi-infarct dementia and is recovering from a fractured hip
following a fall injury. In your initial interview with his wife you would expect to
find:
A. agitation and sundowning.
B. history of steady progression of loss of judgment and poor safety awareness.
C. history of sudden onset of new cognitive problems and patchy distribution of
deficits.
D. perseveration on a thought or activity.

’05 Sullivan A
Practice Questions 39

A129 C1 2
129. A fourteen-year-old girl complains of subpatellar pain after participation in an
aerobic exercise program for two weeks. Your examination shows a large Q
angle, pain with palpation at the inferior pole of the patella, and mild swelling at
both knees. Physical therapy intervention should promote:
A. hamstring strengthening.
B. lateral patellar tracking.
C. vastus lateralis strengthening.
D. vastus medialis muscle strengthening.

A130 C2 3
130. You receive a referral from an acute care physical therapist to treat a patient with
right hemiparesis in the home. The referral indicates that the patient demonstrates
good recovery: both involved limbs are categorized as stage 4 (Brunnstrom
recovery stages). He is ambulatory with an small-based quad cane. The activity
that would be MOST appropriate for a patient at this stage of recovery is:
A. sitting, marching in place (alternate hip flexion movements).
B. standing, picking the foot up behind and slowly lowering it.
C. standing, small range knee extension to gain quadriceps control.
D. supine, bending the hip and knee up to the chest with some hip abduction.

A131 C4 2
131. During a physical therapy session for low back pain, a 67 year-old patient tells
you that she has had urinary incontinence for the last year. It is particularly
problematic when she has a cold and coughs a lot. She has not told her doctor
about this problem because she is too embarrassed. Your BEST course of action
is to:
A. examine the patient and proceed with her back treatment.
B. examine the patient, document and discuss your findings with the doctor.
C. examine the patient, document the problems, then send her back to her doctor.
D. refer the patient back to her doctor.

A132 C2 1
132. During a sensory exam you determine that a patient complains of a dull, aching
pain but is not able to discriminate a stimulus as sharp or dull. Two-point
discrimination is absent. Based on these findings, the pathway that is intact is the:
A. anterior spinothalamic tract.
B. dorsal columns/neospinothalamic systems.
C. fasciculus gracilis/medial lemniscus.
D. lateral spinothalamic tract.

A133 C3 2

‘05 Sullivan A
40 Practice Questions

133. A patient is five days post-MI and is receiving cardiac rehabilitation. At this time
a goal that would be INAPPROPRIATE is to:
A. counteract deconditioning associated with bed rest.
B. educate the patient and family regarding risk factor reduction.
C. increase the patient’s maximal oxygen consumption by discharge.
D. initiate early return to independence in activities of daily living.

A134 C6 3
134. A 73 year-old man has bilateral short transfemoral (AK) amputations and will
require a wheelchair for functional mobility in the home and community. An
appropriate prescription for his wheelchair includes:
A. increasing the seat depth by 2 inches to accommodate the length of the
residual limbs.
B. lowering the seat height by 3 inches.
C. placement of the drive wheels 2 inches anterior to the vertical back supports.
D. placement of the drive wheels 2 inches posterior to the vertical back supports.

’05 Sullivan A
Practice Questions 41

A135 C7 2
135. A multicenter study was done on the reliability of passive wrist flexion and
extension goniometric measurements using volar/dorsal alignment, ulnar
alignment and radial alignment. Significant differences were revealed between
the three techniques. An appropriate level for determining significant difference is
a P value of:
A. P=0.015
B. P=0.05
C. P=0.1
D. P=0.5

A136 C1 3
136. In treating a patient with a diagnosis of right shoulder impingement syndrome, the
therapist should not allow the patient to perform assisted repetitive overhead
exercises without FIRST:
A. controlling all pain.
B. having complete AROM at the shoulder.
C. instruction in proper postural alignment.
D. stretching the shoulder girdle muscles.

A137 C8 3
137. An older person with a diagnosis of degenerative joint disease should understand
his disease, its progression, and its management. You will know your patient
education has been effective if he can tell you:
A. aerobic conditioning is not appropriate when pain is present and medications
must be used.
B. joint protection strategies are important but cannot reduce the expected loss of
function.
C. loss of ROM and immobility are expected and irreversible.
D. pain and stiffness are worse in the early morning and should decrease with
moderate activity.

A138 C2 1
138. A 34 year-old patient is recently admitted to your facility with a diagnosis of
Guillain-Barré syndrome. In your initial examination, you expect to find:
A. asymmetrical weakness with hyperreflexia, bulbar palsy.
B. glossopharyngeal and vagal paralysis with hyperactive jaw and snout reflexes.
C. sensory loss (stocking and glove distribution) with minor loss of motor
function.
D. symmetrical distribution of weakness, ascending with possible involvement of
lower cranial nerves.

A139 C5 3

‘05 Sullivan A
42 Practice Questions

139. A 31 year-old patient presents with patellar tendinitis as a result of a mountain


climbing accident three weeks ago. The patient complains of pain on resisted
knee extension, stair climbing, and sit-to-standing movements. You choose to
apply iontophoresis using a pain medication with a positive charge. The correct
current type, polarity and active electrode placement is:
A. biphasic current with the cathode placed proximal on the tendon.
B. low volt continuous current with the anode placed distal on the tendon.
C. monophasic current with the anode placed on the tendon.
D. monophasic current with the cathode placed on the tendon.

A140 C6 3
140. A 52 year-old patient sustained a T10 spinal cord injury four years ago. During
initial examination you observe redness over the ischial seat that persists for 10
minutes when not sitting. The BEST intervention in this case would be to:
A. change to a low density wheelchair cushion.
B. have the patient do sitting push-ups at least every 10 minutes.
C. increase the arm rest height.
D. order a tilt-in-space wheelchair.

’05 Sullivan A
Practice Questions 43

A141 C1 1
141. A 16 year-old adolescent female was sent to physical therapy with a diagnosis of
anterior knee pain. Positive findings include pes planus, lateral tibial torsion and
genu valgum. The position the femur will be in is excessive:
A. abduction.
B. lateral rotation.
C. medial rotation.
D. retroversion.

A142 C2 3
142. A 24 year-old patient in a coma recovering from traumatic brain injury is
receiving PROM exercises. The family is confused because the occupational
therapist has told them to do range of motion exercises in a different way than the
physical therapist had instructed. As the physical therapist it would be BEST to
deal with this situation by telling the family:
A. the PT and OT supervisors will review and correct any discrepancies in the
exercise program.
B. there is no need for them to worry and note the problem in the patient’s record.
C. to exercise the way you instructed until any differences can be worked out
with the occupational therapist.
D. you will meet with the occupational therapist to discuss the exercise
approaches and you will let them know the outcome of the meeting as soon as
possible.

A143 C7 3
143. You are working with a 10 year-old girl with cerebral palsy. Part of the exercises
in her plan of care involve using the Swiss ball. The choice of educational media
that is BEST to use when instructing her in use of this device is:
A. a oral presentation that uses transparencies of Swiss ball positions.
B. a slide presentation of exercises using the Swiss ball.
C. a videotape of another child with cerebral palsy on a Swiss ball.
D. printed handouts with stick figure drawings and instructions.

A144 C3 3
144. As a physical therapist you are the health professional in charge during a high
school football game. During the game, a player is tackled violently by two
opponents. You determine that the player is unresponsive. Your immediate course
of action should be to:
A. ask for help to log roll the player on his back while stabilizing his neck.
B. open the airway by using the chin-lift method.
C. stabilize the neck and flip back the helmet face mask.
D. summon emergency medical services.

‘05 Sullivan A
44 Practice Questions

A145 C8 2
145. During a home visit, the mother of an 18 month-old child with developmental
delay and an atrio-ventricular shunt for hydrocephalus tells you that her daughter
vomited several times, was irritable and is now lethargic. Your BEST course of
action is to:
A. call the girl’s pediatrician immediately.
B. give the child a cold bath to try and rouse her.
C. give the child clear liquids since she vomited.
D. place the child in a sidelying position and monitor vital signs.

A146 C4 2
146. A 24 year-old woman recently delivered twins on the obstetrical service of your
hospital. After delivery she has developed a 4 centimeter diastasis recti
abdominis. The BEST initial intervention for this problem is to teach:
A. gentle stretching of hamstrings and hip flexors.
B. pelvic floor exercises and sit-ups.
C. pelvic tilts and bilateral straight leg raising.
D. protection and splinting of the abdominal musculature.

’05 Sullivan A
Practice Questions 45

A147 C3 3
147. The optimal position for ventilation of a patient with a C5 complete spinal cord
injury is:
A. semi Fowler’s.
B. sidelying, head of bed elevated 45 degrees.
C. sidelying, head of bed flat.
D. supine, head of bed flat.

A148 C7 2
148. A 16 year-old patient with osteosarcoma is being seen in physical therapy for
crutch training. Her parents have decided not to tell her about her diagnosis. She
is quite perceptive and asks you directly if she has cancer and about her future.
Your BEST course of action is to:
A. change the subject and discuss the plans for that day’s treatment.
B. discuss her condition gently indicating her parent’s fears about not telling her
the diagnosis.
C. schedule a conference with the doctor and family about her condition and your
discussions with the patient.
D. tell the patient that you don’t know the specifics of her condition or prognosis,
and she should speak with her doctor.

A149 C2 2
149. Your patient has had amyotrophic lateral sclerosis for the past two years with
mild functional deficits. He is still ambulatory with bilateral canes but is limited
in his endurance. An important goal for his physical therapy plan of care should
be to prevent:
A. further gait deterioration as a result of ataxia.
B. myalgia.
C. overwork damage in weakened, denervated muscle.
D. radicular pain and paresthesias.

A150 C6 3
150. A 28 year-old patient has extensive full thickness burns to the dorsum of the hand
and forearm. He is to be fitted with a resting splint to support his wrists and hands
in a functional position. An appropriately constructed splint positions the wrist
and hand in:
A. neutral wrist position with IP extension and thumb flexion.
B. neutral wrist position with slight finger flexion and thumb flexion.
C. slight wrist extension with fingers supported and thumb in partial opposition
and abduction.
D. slight wrist flexion with IP extension and thumb opposition.

A151 C3 1

‘05 Sullivan A
46 Practice Questions

151. Your patient is 82 years-old with a long history of congestive heart failure. You
are alert to the signs of left-sided heart failure associated with activity. The
MOST important indicators include:
A. bilateral ankle swelling within 2-3 hours post exercise.
B. complaints of fatigue with increasing dyspnea and cough.
C. nausea and anorexia.
D. sudden weight gain within 24 hours following exercise.

A152 C3 2
152. A patient using an incentive spirometer complains of feeling lightheaded. Your
instructions to the patient should be to:
A. lie down while using the spirometer.
B. take a deeper breath on the following attempt.
C. take a rest period and only use the device 10 times per hour.
D. try to use the spirometer more frequently to get used to it.

’05 Sullivan A
Practice Questions 47

A153 C8 3
153. An eighteen month-old child with Down Syndrome and moderate developmental
delay is being treated at an Early Intervention Program. The schedule that would
best facilitate motor learning of this child is physical therapy intervention given:
A. 30 minutes each day.
B. once a week for two hours.
C. one hour each week with a portion of that time used to teach the caretaker a
home program to be done 3 times a week.
D. three times a week for fifteen minutes.

A154 C1 1
154. A patient has been referred to you for acute shoulder pain after shoveling snow in
a driveway for two hours. Positive findings include pain and weakness with
flexion of an extended upper extremity as well as scapular winging with greater
than 90 degrees of abduction. The patient’s problem is MOST LIKELY the result
of:
A. compression of the long thoracic nerve.
B. compression of the suprascapular nerve.
C. subdeltoid bursitis.
D. supraspinatus tendinitis.

A155 C2 1
155. A patient with multiple sclerosis demonstrates strong bilateral lower extremity
extensor spasticity in the typical distribution of antigravity muscles. You would
expect this patient to demonstrate:
A. sacral sitting.
B. sitting with both legs abducted and externally rotated.
C. sitting with the pelvis tilted, weight bearing on ischial tuberosities.
D. skin breakdown on the ischial tuberosities and lateral malleoli.

A156 C4 1
156. A 58 year-old woman with osteopenia has been on Premarin for the past 4 years.
Based on your knowledge of estrogen replacement therapy, you recognize this
patient is more susceptible to:
A. fractures.
B. osteoporosis.
C. peripheral edema.
D. vasomotor symptoms (hot flashes).

A157 C4 2
157. A 24 year-old pregnant woman who is 12 weeks pregnant asks you if it is safe to
continue with her aerobic exercise. Currently she jogs 3 miles, 3 times a week.
Your BEST answer is:

‘05 Sullivan A
48 Practice Questions

A. continue jogging only until the 5th month of pregnancy.


B. during the last trimester do not jog but switch to exercising in the supine
position only.
C. jogging is safe but you might want to switch to swimming during later months.
D. jogging is safe but your target HR should not exceed 140 beats/min.

A158 C6 3
158. Your patient returns to P.T. after his first exercise session complaining of muscle
soreness that developed later in the evening after his first session and continued
into the next day. He is unsure he wants to continue with exercise. You can
minimize the possibility of this happening again by using:
A. concentric exercises, 3 sets of 10, at 80% of maximal intensity.
B. concentric exercises, 3 sets of 10, with gradually increasing intensity.
C. eccentric exercises, 1 set of 10, lifting body weight (sit-to-stand).
D. eccentric exercises, 3 sets of 10, with gradually increasing intensity.

’05 Sullivan A
Practice Questions 49

A159 C1 1
159. Upon examining a patient with vague hip pain which radiates to the lateral knee,
you have found a negative FABERE test, negative grind test, and a positive Noble
compression test. The dysfunction is most likely due to:
A. a possible fracture of the femoral neck.
B. an iliotibial band friction disorder.
C. degenerative joint disease of the hip.
D. sacroiliac joint dysfunction.

A160 C2 1
160. A 22 year-old male suffered carbon monoxide poisoning from a work-related
factory accident. He is left with permanent damage to his nervous system,
affecting the basal ganglia. You logically expect his symptoms to include:
A. impaired sensory organization of balance.
B. motor paralysis.
C. muscular spasms and hyperreflexia.
D. problems with motor planning and scaling of movements.

A161 C2 1
161. A patient presents with problems with swallowing. When you test for phonation
by having the patient say “AH” with his mouth open, you notice there is deviation
of the uvula to one side. You then test for function of the gag reflex and notice
loss of response to stimulation. These findings suggest involvement of the:
A. facial nerve.
B. hypoglossal nerve.
C. trigeminal nerve.
D. vagus nerve.

A162 C3 1
162. A patient with COPD is sitting in a bedside chair. The apices of the lungs in this
position compared with other areas of the lungs in this position would
demonstrate:
A. increased perfusion.
B. increased volume of air at resting end expiratory pressure (REEP).
C. the highest changes in ventilation during the respiratory cycle.
D. the lowest oxygenation and highest CO2 in blood exiting this zone.

A163 C6 1
163. Checkout for a lower limb orthosis includes inspection of the alignment of
anatomic and orthotic joints. During a sagittal plane check-out you determine that
the orthotic hip joint is malaligned. The correct position is:
A. 3 inches below the anterior superior iliac spine.
B. just anterior and superior to the greater trochanter.

‘05 Sullivan A
50 Practice Questions

C. just posterior and inferior to the greater trochanter.


D. lateral to the greater trochanter.

A164 C7 3
164. A physical therapist assistant you supervise treated a patient in the home care
setting. The patient is status post CVA. Part of the plan of care includes
“progressive gait training on level surfaces”. The patient falls and sustains a
fractured hip during a visit done by the PTA. The fall occurred when the PTA
took the patient on the stairs for the first time. The responsible party in this case
is:
A. both the PT and the PTA because the PT gave inadequate supervision, and the
PTA used poor judgment.
B. neither the PT nor the PTA because patients who have sustained a CVA are
always at high risk for falling, and thus it is a regrettable occurrence only.
C. the PT who is negligent for failing to provide adequate supervision of the
PTA.
D. the PTA who is completely liable because the plan of care was altered without
communicating with the supervising PT.

’05 Sullivan A
Practice Questions 51

A165 C1 3
165. As the result of blunt trauma to the quadriceps femoris muscle, a patient
experiences loss of knee function. Early PT interventions should stress:
A. aggressive open-chain strengthening of the quadriceps femoris to regain
normal lower extremity strength.
B. aggressive soft tissue stretching to remove blood which has accumulated in
soft tissues.
C. gentle AROM exercises in weight bearing.
D. gentle PROM exercises in nonweightbearing to regain normal knee motion.

A166 C8 3
166. An important adjunct to physical therapy management of a child with moderate
spastic hemiplegia would be use of:
A. a KAFO on the affected side.
B. a posterior walker.
C. a tone inhibiting ankle-foot orthosis (AFO).
D. an anterior rollator walker.

A167 C2 1
167. While evaluating the gait of a patient with right hemiplegia, you note foot drop
during midswing on the right. The MOST LIKELY cause of this deviation is:
A. decreased proprioception.
B. excessive extensor synergy.
C. excessive flexor synergy.
D. inadequate contraction of the ankle dorsiflexors.

A168 C7 3
168. A patient’s daughter wants to look at her father’s medical record. He has recently
been admitted for an insidious onset of low back pain. You, as the physical
therapist, should:
A. give her the chart and let her read it.
B. tell her she cannot see the chart because she could misinterpret the
information.
C. tell her that she must have the permission of her father before she can look at
the chart.
D. tell her to ask the physician for permission

A169 C5 2
169. A 48 year-old female has had a total knee replacement. Following surgery, you
place her on a regimen of continuous passive motion. The primary purpose for
applying CPM for the first few hours is to:
A. decrease edema.
B. decrease pain.

‘05 Sullivan A
52 Practice Questions

C. increase range of motion.


D. increase tissue tensile strength.

A170 C5 2
170. A 37 year-old male developed acute bicipital tendinitis two days after water
skiing. He is unable to work as a result of the pain he rates as 8/10. He is referred
to physical therapy for iontophoresis treatment to help relieve the pain. For the
first two weeks this patient should optimally be seen:
A. five times per week
B. once a week.
C. three times per week.
D. two times per week

A171 C6 1
171. You observe genu recurvatum as a patient with hemiplegia ambulates using his
posterior leaf spring (PLS) orthosis. A factor that is NOT a possible cause of this
gait deviation is:
A. extensor spasticity.
B. hip flexion contracture.
C. pes equinus.
D. quadriceps weakness.

’05 Sullivan A
Practice Questions 53

A172 C3 2
172. During a home visit you are providing postural drainage in the Trendelenburg
position to a 15 year-old male with cystic fibrosis. The patient suddenly
complains of right-sided chest pain and shortness of breath. On auscultation, there
are no breath sounds on the right. The physical therapist should:
A. call emergency medical technicians as it may be a pneumothorax.
B. continue treating as it is possibly a mucous plug.
C. place the right lung in a gravity dependent position to improve perfusion.
D. reposition patient in the head of bed flat position as Trendelenburg is causing
shortness of breath.

A173 C2 3
173. A 65 year-old patient with multiple sclerosis is being treated at home. The patient
is bedridden for most of the day with only short periods up in a bedside chair.
Medicare is funding the patient’s home care program which has as its primary
goals maintaining PROM and positioning to prevent deformity. Your role as the
physical therapist is to provide:
A. a limited cardiovascular conditioning (sitting) program aimed at improving
respiratory capacity.
B. a restorative exercise program aimed at improving upright sitting control and
improved functional independence.
C. PROM exercises 2 times a day with additional family instruction to ensure
weekend coverage.
D. supervision of home health aides for completion of a daily home exercise
program.

A174 C2 3
174. Your patient is a 72 year-old man with an 8 year history of Parkinson’s disease.
He demonstrates significant rigidity, decreased PROM in both upper extremities
in the typical distribution, and frequent episodes of akinesia. The exercise that
BEST deals with these problems would be:
A. modified plantigrade, isometric holding, stressing upper extremity shoulder
flexion.
B. PNF bilateral symmetrical upper extremity D2 flexion patterns, rhythmic
initiation.
C. quadruped position, upper extremity PNF D2 flexion and extension.
D. resistance training, free weights for shoulder flexors at 80% of 1 repetition
max.

A175 C7 3
175. A 72 year-old patient with diabetes is recovering from recent surgery to graft a
large decubitus ulcer over the heel of her left foot. You are concerned that her
loss of range at the ankle (-5° to neutral) will limit her ambulation and

‘05 Sullivan A
54 Practice Questions

independent status. One afternoon you are very busy and request that one of the
physical therapy aides do her range of motion exercises. The aide is new to your
department but tells you she is willing to take this challenge on if you show her
how to do it. Your BEST course of action is to:
A. modify the treatment session to eliminate the ROM exercises and have the
aide walk the patient in the parallel bars.
B. perform the ROM exercises yourself.
C. reschedule the patient for tomorrow when you have more time.
D. take 5 minutes to instruct the aide in ROM exercises, then have her do them.

A176 C8 2
176. The MOST appropriate type of adaptive equipment for a teenager with severe
athetosis and severe extensor spasms of the lower extremities would be:
A. posterior rollator walker.
B. scooter board.
C. supine stander with head support.
D. wheelchair with head and trunk support with abductor wedge.

A177 C1 2
177. The symptoms of ankylosing spondylitis in its early stages can best be managed
in physical therapy by:
A. back extension, costal expansion exercises, and maintenance of proper posture
to prevent deformity.
B. cardiovascular conditioning, weight bearing exercises, and joint protection
education.
C. pain management, abdominal strengthening, and breathing exercises.
D. strengthening of anterior chest muscles, costal expansion, and stretching of
scapular stabilizers.

’05 Sullivan A
Practice Questions 55

A178 C2 1
178. A 72 year-old male was referred for rehabilitation following a middle cerebral
artery stroke. Based on this diagnosis you suspect he will present with:
A. aphasia since his right hemisphere is involved and he is right hand dominant.
B. contralateral hemiparesis and sensory deficits with greater involvement in the
arm than the leg.
C. contralateral hemiparesis and sensory deficits with greater involvement of leg
than the arm.
D. homonymous hemianopsia, contralateral hemiplegia with thalamic sensory
syndrome.

A179 C3 1
179. The cardiac rehabilitation team is conducting education classes for a group of
patients. The focus is on risk factor reduction and successful life style
modification. A participant asks you which is the bad cholesterol that increases
atherosclerosis and risk of heart disease. Your answer is:
A. all cholesterol is bad and total levels should be below 200 mg/dl.
B. cholesterol is not the important factor; the triglycerides should be kept low.
C. elevated levels of high-density lipoprotein or HDL.
D. elevated levels of low-density lipoprotein or LDL.

A180 C7 3
180. An attractive physical therapist is treating a young and handsome football player
with an ACL sprain. She is very fond of this patient and enjoys treating him.
After a few visits, the football player asks her out to dinner. The physical
therapist’s response should be to:
A. thank him very much, and accept his offer for dinner.
B. thank him very much, and invite him for dinner at her apartment with other
guests.
C. thank him very much, but refuse his invitation while he is receiving treatment.
D. transfer the patient care to one of her colleagues and then go out to dinner with
him.

A181 C1 2
181. After running one mile, an athlete complains of deep cramping at the anterior
aspects of the legs which does not ease and prevents continued running.
Management of this problem should include:
A. orthotic fabrication to enable continued running on all surfaces
B. referral to a physician to evaluate anterior compartment pressures during
activity.
C. referral to a physician to rule out spinal stenosis.
D. stretching of the tibialis anterior muscles to help resolve shin splints.

‘05 Sullivan A
56 Practice Questions

A182 C2 2
182. A 62 year-old woman developed polio at the age of 6 with significant lower
extremity paralysis. She wore bilateral long leg braces for a period of 2 years. She
then recovered enough to stop using her braces but still required bilateral
Lofstrand crutches, then bilateral canes to ambulate. Recently she has been
complaining of new difficulties (she has had to start using her crutches again).
You suspect post-polio syndrome. The BEST goal/intervention for this patient
based on her current findings is to:
A. prescribe an aquatic therapy program consisting of daily 1 hour aerobics.
B. put her on a lower extremity resistance training program utilizing 80% one
repetition max.
C. teach her activity pacing and energy conservation techniques.
D. utilize a moderate conditioning program consisting of cycle ergometry 3 times
a week of 60 minutes at 60% maximal heart rate.

A183 C1 1
183. A patient diagnosed with lumbar spondylosis without discal herniation or bulging
has a left L5 neural compression. The most likely structure compressing the nerve
root is:
A. anterior longitudinal ligament.
B. ligamentum flavum.
C. posterior longitudinal ligament.
D. supraspinous ligament.

’05 Sullivan A
Practice Questions 57

A184 C3 2
184. During an exercise tolerance test (ETT) a patient demonstrates poor reaction to
increasing exercise intensity. According to the American College of Sports
Medicine, an absolute indication for terminating this test is:
A. 1.5 mm of downsloping ST-segment depression.
B. fatigue and shortness of breath.
C. onset of moderate to severe angina.
D. supraventricular tachycardia.

A185 C2 3
185. A patient with T10 paraplegia is receiving daily ROM exercises. On this day you
notice swelling, pain, local warmth and erythema in the thigh near the hip joint
with some limitation in motion. Your BEST course of action is:
A. apply ice before his ROM exercises to ease his pain and discomfort and
inflammation.
B. document the findings in his record and double his daily sessions of ROM
exercises.
C. instruct his aides to be more vigorous in positioning and ROM exercises.
D. notify the physician immediately, you suspect heterotopic ossifications may be
developing.

A186 C6 1
186. A patient with a T4 spinal cord injury is being measured for a wheelchair. In
determining the correct seat height you can use as a measure:
A. clearance between the floor and the foot plate of at least 2 inches.
B. clearance between the floor and the foot plate of at least 4 inches.
C. the distance from the bottom of the shoe to just under the thigh at the popliteal
fossa.
D. the patient’s leg length measurement plus four inches.

A187 C8 2
187. A 6 year-old boy has a diagnosis of Duchenne muscular dystrophy and is still
ambulatory. The MOST appropriate activity to include in his plan of care would
be:
A. circuit training program.
B. progressive resistance strength training.
C. recreational activities such as swimming or biking.
D. wheelchair sports.

A188 C7 3
188. A 72-year old patient is receiving outpatient physical therapy at your private
clinic. The clinic is an approved Medicare outpatient provider. Your patient is

‘05 Sullivan A
58 Practice Questions

concerned that she will not be able to pay for her continuing care and worries that
her Medicare benefits will run out soon. You tell her:
A. coverage is limited only for hospital-based outpatient PT services.
B. currently there is no limit to her Medicare coverage for outpatient PT services.
C. there is currently a limit of $1000.00 for coverage of outpatient PT services.
D. there is currently a limit of $1590.00 for coverage of outpatient PT services.

A189 C2 3
189. Your patient is a 42 year-old woman who suffered a stroke and demonstrates a
locked-in state characterized by spastic quadriplegia and bulbar palsy. To
facilitate communication with this patient you should instruct the family to:
A. give her a chance to mouth her responses even though she can’t vocalize well.
B. look closely at her facial expression to detect signs of what she is trying to
communicate.
C. use a communication board with minimal movements of her hand.
D. use an alternate eyelid taping schedule which will allow her to use eye
movements to communicate.

A190 C6 2
190. After performing an ergonomic examination of a worker and workstation the
most appropriate recommendation for achieving ideal wrist and elbow positioning
would be to:
A. add armrests.
B. elevate the keyboard to increase wrist flexion.
C. lower the keyboard to increase wrist extension.
D. maintain the keyboard in a position allowing a neutral wrist position.

A191 C3 1
191. A 43 year-old patient with a post myocardial infarction is on digitalis to improve
cardiac contractility. He is a new participant in your Phase 2 outpatient cardiac
rehabilitation program. He is being continuously monitored by ECG via radio
telemetry. On his ECG, the medication induced changes that could be expected
are:
A. decreased heart rate with prolonged QRS and QT intervals.
B. depressed ST segment with a flat T wave and shortened QT interval.
C. elevated ST segment with T wave inversion.
D. widened QRS complex with a flattened P wave.

A192 C3 2
192. Your patient has lymphatic disease of the right arm secondary to radical
mastectomy and radiation. The resulting edema can BEST be managed in
physical therapy by:
A. AROM and extremity positioning.

’05 Sullivan A
Practice Questions 59

B. intermittent pneumatic compression, extremity elevation, and massage.


C. isometric resistive exercises and extremity positioning in elevation.
D. PROM and extremity elevation.

A193 C7 2
193. You have been treating a patient for chronic subluxation of the patella in the
outpatient clinic. He is now scheduled for a lateral release and is worried about
any complications of the surgical procedure. He asks you to describe any
potential complications. Your BEST response is to:
A. do a search on the internet and get back to the patient with the desired
information.
B. explain how patients you have treated responded to the surgery.
C. refer the patient to a physical therapy colleague who specializes in knee
problems.
D. suggest that the patient speak with his surgeon.

A194 C1 3
194. A 46 year-old female was referred for physical therapy following a right breast
lumpectomy with axillary lymph node dissection. Scapular control is poor when
upper extremity flexion or abduction is attempted. PT intervention should focus
on:
A. active assistive pulley exercises to assist rotator cuff muscles as a result of
damage to the suprascapular nerve.
B. gravity assisted right upper extremity exercises to promote scapular control
following damage to the long thoracic nerve.
C. rhomboid strengthening as a result of disuse of the scapular stabilizers.
D. strengthening of the right deltoids to help stabilize the shoulder to compensate
for damage to the dorsal scapular nerve.

A195 C3 3
195. A post surgical patient is receiving a regimen of postural drainage three times a
day. You could reduce the frequency of treatment if the:
A. amount of productive secretions decreases.
B. consistency of the sputum changes.
C. patient becomes febrile.
D. patient experiences decreased postoperative pain.

A196 C2 1
196. The distinguishing feature of Mèniére’s disease which sets it apart from other
peripheral conditions affecting the vestibular system is:
A. chronic, episodic bouts of dysfunction.
B. postural instability.
C. significant dysfunction with severe attacks of vertigo and nausea.

‘05 Sullivan A
60 Practice Questions

D. time-limited, usually lasting only 24-48 hours.

’05 Sullivan A
Practice Questions 61

A197 C7 3
197. A physical therapist arriving at work one hour before she was due to start work,
began moving treatment tables and rearranging the physical therapy clinic. This
operation could have been done during regular hours. The therapist sustained a
low back injury as a result of moving the equipment. Payment for the therapist’s
care relating to this incident would be covered primarily by:
A. employee’s health insurance.
B. the hospital’s insurance company.
C. the therapist’s own resources.
D. Workers’ Compensation.

A198 C1 2
198. Patients diagnosed with Paget’s disease typically have similar symptomatology to
spinal stenosis. The most important aspect of physical therapy intervention
emphasizes:
A. lumbar extension exercises.
B. modalities to decrease pain.
C. postural reeducation to prevent positions that increase symptoms.
D. strengthening exercises for the abdominals and back muscles.

A199 C6 1
199. A patient with spastic left hemiplegia experiences severe genu recurvatum during
stance phase. If the patient is using an ankle-foot orthosis, the cause of the
problem might be attributed to:
A. the anterior stop setting the foot in too much dorsiflexion.
B. the anterior stop setting the foot in too much plantar flexion.
C. the posterior stop setting the foot in too much dorsiflexion.
D. the posterior stop setting the foot in too much plantarflexion.

A200 C3 3
200. A patient has arterial peripheral vascular disease with symptoms of intermittent
claudication in both lower extremities. Which of the following is NOT an
appropriate guideline for exercise training?
A. daily walking through pain in order to attain predetermined distances.
B. daily walking, 2 to 3 times a day.
C. interval training protocol with frequent rests.
D. stopping exercise and resting once the pain threshold is reached

‘05 Sullivan A
62 Practice Questions

1. glenohumeral mobilization and strengthening of scapular stabilizers to regain


normal scapulohumeral movement. Compensation of glenohumeral restrictions if
often exhibited as excessive scapular movement. Therefore, mobilization of the
glenohumeral joint and strengthening of scapular stabilizers is needed to regain
normal scapulohumeral motion.

2. unilateral hearing loss. Presbycusis is a sensorineural hearing loss associated


with aging. It is characterized by bilateral hearing loss with deficits noted in
auditory discrimination and comprehension.

3. increase the warm-up and cool-down periods to equal the total aerobic interval
in time. Clinical decisions should focus on reducing the environmental costs of
exercising (change the time of the day of the exercise class to reduce the heat
stress) or reducing the overall metabolic costs of the activity (decrease the pace of
the exercise, add more rest periods). Altering the warm-up and cool-down periods
does not lower the overall cost of the aerobic exercise period.
4. anterior tibia, tibial crest, and fibular head. In the PTB socket, reliefs are
provided for pressure sensitive areas: the anterior tibia and tibial crest, fibular
head, and peroneal nerve. All the other choices are considered pressure tolerant
areas.

’05 Sullivan A
Practice Questions 63

5. take time now to allow the patient to express her fears and frustrations. It is
important to be supportive of a patient who is experiencing losses and resentment.
Allow the patient to fully verbalize her feelings and frustrations.

6. 1 MHz continuous at 1.0 W/cm2. 1 MHz of continuous ultrasound provides deep


heating to a depth of 3-5 cm. At this frequency, attenuation (absorption) is les in
superficial tissues. His allows more energy to be absorbed, thus more heat
production in deeper tissue layers. Continuous US is applied to achieve thermal
effects (i.e., chronic pain) and pulsed US is used when nonthermal effects are
desired (i.e., acute soft tissue injuries).

7. some recovery of function since damage is to peripheral nerve roots. A spinal


cord lesion below L1 is a cauda equina lesion (injury to the peripheral roots and
nerves). Since some regeneration is possible, some recovery in function can be
expected. A spastic or reflex bladder is associated with upper motor neuron
injury. Other choices describe the deficits associated with anterior cord syndrome
or central cord syndrome.

8. be designated as a more advanced task and more appropriately delegated to


another physical therapist. Physical therapy students should not perform
advanced tasks for the first time without any instruction or direct supervision.
This might be unsafe for the patient. The task of orthotic checkout is most
appropriately performed by another physical therapist.

9. instruction to eat a soft food diet and phonophoresis. Phonophoresis and


education regarding consumption of only soft food with help resolve the acute
inflammatory process in the temporomandibular joint. Application of an intraoral
appliance occurs only when the acute inflammation is not resolved or bruxism
continues. Joint mobilization should not be attempted with an acute inflammation.

10. posterior right C7 articular pillar. The most effective hand placement for
mobilization into great left rotation is at the posterior aspect of the right C7
articular pillar because it rotates the C7 vertebra to the left.

11. transfer objects from one hand to another. Transferring objects from one hand to
another is a task developmentally appropriate for an 8-9 month-old. Using a fine
pincer grasp and building a tower of 4 blocks are skills which develop later.
Holding a cup by the handle while drinking usually occurs by 12 months of age.

12. document your observations and refer him back to his physician for evaluation of
possible dopamine toxicity. Dyskinesias (involuntary movements) are a common
side effect of dopamine toxicity as are gastrointestinal disturbances (nausea,

‘05 Sullivan A
64 Practice Questions

vomiting) and mental disturbances (restlessness, general overactivity, anxiety or


depression). While the symptoms described in the other choices may also occur
with Parkinson’s disease, they do not adequately explain the presence of
adventitious or involuntary movements.

13. explain how patients typically respond to the surgery and outline the progression
of exercises. Assess the needs of the patient and provide appropriate information
based on the expected rehabilitation process. Do not “pass the buck” unless the
information is outside of the scope of your expertise.

14. patient reaches age-predicted maximal heart rate. A maximum exercise


tolerance test is a sign or symptom limited test. Achieving age adjusted predicted
maximum heart rate is not a sign or symptom and therefore does not stop the test.

15. swing-to. A swing-to gait pattern is indicated to individuals with limited use of
both lower extremities and trunk instability. It is slower and more stable than a
swing-through gait pattern (a gait pattern this patient can be progressed to after
his initial training). This patient is unable to perform a reciprocal gait.

16. request her primary physician to refer her for psychological evaluation. The
patient is experiencing grief over her loss. Significant persistent symptoms are an
indication for a referral to a qualified professional (psychologist) to help her deal
with her loss.

17. friction. Friction massage is effective when applied across (perpendicular)


muscle fibers to stretch scars and loosen adhesions in the tissue due to the
inflammatory process. Stroking is a relaxation technique and usually initiates and
ends massage treatment. Some passive muscle stretching is performed with deep
stroking. Kneading aids in loosening adhesions and increasing venous return.
Tapotement is used when nerve stimulation or lung decongestion is the desired
treatment effect.

18. vigorous range of motion of the lower extremities. Vigorous range of motion is
contraindicated due to osteoporosis and risk of fracture. All of the other choices
are appropriate to promote functional independence.

19. SI joint. Pain at end range of flexion, abduction, and external/lateral rotation
(FABERE Test) is diagnostic for SI joint dysfunction because it both gaps and
compresses the joint. Pain at the midrange into hip flexion, abduction and
external/lateral rotation suggests hip joint pathology.

’05 Sullivan A
Practice Questions 65

20. chi square test. Chi square is nonparametric statistical test used to compare data
in the form of frequency counts (in this example frequencies of carpal tunnel
syndrome) occurring in two different groups of workers.

21. median. The mean is a measure of central tendency that is calculated by adding
up all the scores and dividing the total by the number of scores. The median is a
middle score while the mode is the most frequently occurring score. The most
accurate measure of performance is skewed distribution with extreme scores in
the median.

22. denervation atrophy has occurred. Spontaneous fibrillation potentials present on


EMG are evidence of denervation atrophy. Polyphasic motor units of low
amplitude and short duration are evidence of reinnervation.

23. one-arm drive chair. A one-arm drive wheelchair has both drive mechanisms
located on one wheel. The patient can propel the wheelchair by using one hand(in
this case his sound left hand). It is contraindicated in patients with cognitive or
perceptual deficits. A hemiplegic chair (low seat height) allows use of both the
sound hand and leg to propel the chair. The electric wheelchair with joystick
might also work but is significantly more expensive, less transportable, and
would require increased maintenance. An elevating legrest is needed to complete
the wheelchair prescription.

24. functional residual capacity. Muscular dystrophy will alter the muscles’ ability
to pull in air and blow out air; therefore, vital capacity, total lung capacity, and
forced expiratory volume in one second will be decreased. Since muscular
dystrophy does not change the lung parenchyma, REEP will occur at same point
of equilibrium between lung recoil and thoracic outward pull. Therefore,
functional residual volume will not change.

25. swing phase. Foot drop is a swing phase deficit. Stimulation of the dorsiflexor
muscles during the swing phase places the foot in a more neutral position and
prevents the toes from contacting the ground and interfering with the gait pattern.

26. refuse to treat that patient. Blood and Body Fluid Precautionary Guidelines from
the Centers of Disease Control (CDC) state that a healthcare worker with
exudative lesions or weeping dermatitis should refrain from all direct patient care
and from handling patient-care equipment until the condition resolves.

27. tight iliopsoas muscle. Tight iliopsoas muscles are common after THR surgery
because the patient typically spends more time in a sitting position. The iliopsoas
attaches to the transverse processes of the lumbar spine and the lesser trochanter

‘05 Sullivan A
66 Practice Questions

of the femur. Therefore, when the iliopsoas is tight, the lumbar spine will extend
if the femur is fixed. In supine, the hamstrings and piriformis muscles are on
slack and would not cause alteration in spinal position.

28. environmental changes, a bedside commode, and referral for home health
services. Clinical decision making in this case should focus on the patient’s
ability to manage in the home. Environmental modifications (the addition of a
commode) and assistance of a home care aide should allow her to safely return
home. The referral for home physical therapy should focus on improving her
endurance to regain independence in the home. Treatment at home is the most
cost effective in this case.

29. make the patient aware of his deficit and teach him to turn his head to the
affected left side. A patient with homonymous hemianopsia needs to be made
aware of his deficit and instructed to turn his head to the affected left side (a
compensatory training strategy). Initial strategies include placing items or
doorway on his right (unaffected side) so he can successfully interact with his
environment. Later, as he demonstrates the ability to compensate, items can be
moved to midline and finally to his affected left side.

30. reduce the exercise intensity and provide relaxation strategies. Common
problems in multiple sclerosis include fatigue and heat tolerance. The agitation
and irritation are most likely a result of frustration and fatigue. Exercise intensity
should be reduced and relaxation strategies utilized. A cool environment is
important to reduce fatigue but would not address the problems of agitation and
irritation. A warm pool is contraindicated.

31. Medicare. Custodial care of a medically stable individual (over 65) who needs
assistance with basic activities of daily living such as eating, dressing, toileting
and bathing in the home is covered by Medicare only when skilled care (the
services of a professional nurse or therapist under a physician-authorized plan of
home care) is required.

32. instruct in proper dressing changes and wound care. Proper dressing changes
and wound care to prevent secondary wound infection are the hallmarks of
conservative management of venous ulcers. Pliable, nonstretchable dressing
wraps (e.g., Unna boot) or custom-fitted support stockings can be used to assist in
venous circulation while elastic wraps do little to assist circulation. Prolonged
hydrotherapy is contraindicated for venous ulcers.

33. spasticity or contracture of the plantarflexors. Forward advancement of the tibia


from midstance to heel-off is controlled by eccentric contraction of the

’05 Sullivan A
Practice Questions 67

plantarflexors; from heel-off to toe-off the plantarflexors contract concentrically.


Either spasticity or contracture of the plantarflexors would limit this forward
progression. Patients compensate by going right into swing, typically with
circumducted gait, or with increased hip and knee flexion since there is no push
off.

34. thumb abduction. Thumb abductors are innervated by the median nerve,
primarily by the C6 nerve root. The anterior divisions contribute to nerves that
primarily serve flexors, and in this case, the thumb.

35. be calm and supportive, using only one or two level commands. An agitated
patient with dementia does not process information easily. A calm and supportive
approach with low level commands (one or two actions) provides the best
approach to take with this patient.

36. postural hypotension. Fall risk is significantly increased with certain


medications. Tricyclic antidepressants (e.g., Elavil) can be effective in relieving
depression but may cause postural hypotension, fainting or confusion, thus
increasing fall risk. The elderly are particularly susceptible to adverse drug effects
due to multitude of factors.

37. weakness and palpitations. Digitalis (Digoxin) is frequently used to treat


congestive heart failure (it slows heart rate and increases force of myocardial
contraction). Adverse side effects of digitalis can include muscle weakness and
supraventricular or ventricular arrhythmias including ventricular fibrillation
without premonitory signs.

38. a treadmill purchase. Any purchases of $300 or more would be considered a


capital expense and listed as an expense in the capital budget, not an operating
budget.

39. wear gloves if there is direct contact with blood or body fluids. Standard
Precautions specify health care workers wear gloves when they come into direct
contact with blood or body fluids. Health care workers should wear moisture-
resistant gowns and masks for protection from the splashing of blood, other body
fluids or respiratory droplets.

40. continuous monophasic current with the medication under the anode. Continuous
unidirectional current flow is more effective in repelling ions into skin than pulsed
or bidirectional current. Procaine is a positive medicinal ion and will be repelled
from the anode (positive pole).

‘05 Sullivan A
68 Practice Questions

41. long term corticosteroid therapy. Very often patients with chronic pulmonary
disease have been managed using corticosteroid therapy. Long term steroid use
affects ligamentous integrity, which often produces joint hypermobility.

42. Medicaid Long term care for institutionalized elderly is funded by Medicaid, a
state-federal partnership of funding. Patients must first spend down their assets to
qualify for low income stains before they are eligible for Medicaid. Individual slates
determine eligibility requirements.

43. infected pleura. The case is supportive of a pulmonary process as evidenced by


radiography and history. Since the radiographic findings and the pain are in the same
vicinity, the likelihood is this pain is pleuritic in origin. Angina is not the most likely
cause since the cardiac system is not involved. There is no history of trauma to the
chest and no trauma was found radiographically, making it unlikely as the source of
pain. An inflamed tracheobronchial tree would not usually reflect pain in the
posterior base of the left thorax.

44. training all staff to do simple repairs on all electrical equipment if a breakdown
should occur. Electrical equipment is repaired by the manufacturer or local
vendor, or in some cases, the maintenance department, not by physical therapy
staff.

45. cellulitis. Cellulitis is an inflammation of the cellular or connective tissue in or


close to the skin. It is characterized by skin that is hot red, and edematous. Fever
is a common finding. Dermatitis produces red, weeping, crusted skin lesions, but
is not commonly accompanied by fever. Location on shins makes herpes an
unlikely choice and there are no skin eruptions, or vesicles. Scleroderma is a
collagen disease producing tight, drawn skin.

46. ultrasound. Ultrasound energy may affect the epiphyseal areas causing bone
growth disturbances. The epiphyseal plates close at the end of puberty.

47. 6-8 weeks of training. Hypertrophy is the increase in muscle size as a result of
resistance training and can be observed following at least 6-8 weeks of training.
Individual muscle fibers are enlarged, contain more actin and myosin, and have
more, larger myofibrils.

48. external/lateral rotation and extension. The long head of the biceps is best
exposed in shoulder lateral rotation and extension due to its attachment at the
supraglenoid tubercle of the scapula which is at the medial aspect of the shoulder
joint. Medial rotation and abduction places the long head of the biceps deep to the

’05 Sullivan A
Practice Questions 69

anterior deltoid and pectoralis major muscles. The anterior surface of the shoulder,
including the long head of the biceps, loses exposure with horizontal adduction.

49. step up onto a low step while in the parallel bars. Active practice of stepping
using a low step represents the best choice to ensure motor learning. Passively
bringing the loot up does not promote active learning. Choices A and C are
appropriate lead-up skills to stair climbing.

50. tandem walking and single limb stance. A patient with balance instability would
benefit from all of the interventions except tandem walking and single limb
stance. These activities are too difficult (high level) to be included in an initial
training program. A significant percentage of healthy elderly are unable to stand
on one leg.

51. 252 inches (21 feet). The architectural standard for rise of a step is 7 inches (steps
may vary from 7-9 inches). The recommended ratio of slope to rise is 1:12 (an 8%
grade). For every inch of vertical rise, 12 inches of ramp will be required. A
straight ramp will have to be 252 inches or 21 feet long.

52. increased tidal volume. Asymmetrical breathing, deviated trachea towards the
side to the pneumonectomy, and decreased breath sounds would be likely changes.
An increased tidal volume due to incisional pain and sedation would not be likely.

53. shortness of breath at rest and with limited activity, and sudden weight gain.
Elderly patients with moderate to severe congestive heart failure demonstrate
diminished cardiac reserve and intolerance to strenuous physical activity.
Symptoms of exertional intolerance include shortness of breath and sudden
weight gain, the result of circulatory backflow and peripheral edema.

54. position in prone lying and sitting with full knee extension. The typical
contractures with a transtibial amputation are knee flexion and hip flexion
(typically from too much sitting). When in bed, hip and knee extension should be
emphasized (e.g. prone-lying). When sitting in the wheelchair, knee extension should
be emphasized (e.g. using a posterior splint or knee board).

55. postpone therapy and coordinate with the nurse regarding insulin management
and exercise. Normal lasting plasma glucose is less than I 15 mg/dL while a
fasting plasma glucose level greater than 126 mg/dL on more than one occasion
is indicative of diabetes. This patient is hyperglycemic with high glucose levels
(equal to or greater than 250 mg/dL). Clinical signs that may accompany this
condition include ketoacidosis (acetone breath) with dehydration, weak and rapid
pulse, nausea/vomiting, deep and rapid respirations (Kussmaul’s respirations),

‘05 Sullivan A
70 Practice Questions

weakness, diminished reflexes, and paresthesias. The patient may lie lethargic and
confused and may progress lo diabolic coma and death if not treated promptly
with insulin. Physical therapy intervention is contraindicated; exercise can lead to
further impaired glucose uptake.

56. splinting the shoulder in abduction and internal rotation. Splinting the shoulder
in abduction leads to formation of abduction contractures and later hypermobility
of the shoulder.

57. extension, left side-bending, and right rotation. The right sternocleidomastoid
produces left lateral rotation and flexion of the cervical spine. The right
sternocleidomastoid is in lengthened position with the head turned to the right and
the cervical spine extended.

58. he should be expected to value patient confidentiality. Valuing and upholding


patient confidentiality is an expected behavior for all physical therapists and physical
therapy students specified by the American Physical Therapy Association Code of
Ethics. Since this a final affiliation, the student should adhere fully to the APTA's
Code of Ethics.

59. B is less than A. In an A-B-A-B single subject design, the As represent multiple
baseline-measurements while the Bs represent multiple post-treatment
measurements. If the hypothesis is accepted, the pain rating scores will he lower
following the treatment when compared to the baseline measurements.

60. improve rate control at faster movement speeds. Patients during the later stages
of recovery from stroke frequently exhibit problems with rate control. They are
able to move at slow speeds but as speed of movement increases, control decreases.
An isokinetic device can be an effective training modality to remediate this
problem.

61. use a shrinker. A shrinker is a suitable alternative to elastic wraps. It is


important to select the right size shrinker to limit edema and accelerate healing.
An Unna's paste dressing is applied at the time of initial surgery. Use of a
temporary prosthesis should be a prosthetic team decision and is based on
additional factors such as age, balance, strength, cognition and so forth.

62. 5 times/week, BID 10 minutes/session. Patients with vascular insufficiency and


claudication pain should be encouraged to walk daily, 2-3 x/day. Duration should be
short. The patient should walk to the point of maximum tolerable pain, and
allowed to rest.

’05 Sullivan A
Practice Questions 71

63. oblique. The spinal defect is spondylolisthesis and the radiographic view that
demonstrates the scotty dog neck fracture is the oblique view. A lateral
radiographic view will show the degree of anterior or posterior slippage of one
vertebra on another allowing the radiologist to grade the spondylolisthesis.

64. allow the patient to express her anger while refocusing her on effective coping
strategies. Anger is a recognized stage in the psychological adaptation to death
and dying (Kubler-Ross). Patients should be allowed to express anger, frustration,
and resentment. A helpful strategy is to redirect the patient to achieve effective
coping strategies (anger management techniques). While honest, accurate
information is important it is not the most useful strategy for this patient at this
time.

65. a facet joint. Facet joint dysfunction is exacerbated with sustained positions and
eases with movement.

66. document the findings and immediately inform the patient’s physician about the
situation. It is most appropriate to discuss a patient problem with the person who
is ultimately responsible for the patient. The physician as team leader can affect
a change in behavior or seek additional help for the patient.

67. forward step-ups in standing using graduated height steps. Decreased foot
clearance during swing may result from weak hip and knee flexors or may be the
result of a drop foot (weak dorsiflexors or spastic plantarflexors). Step-ups
represents the best choice to functionally strengthen the hip and knee flexors.
Bridging promotes knee flexion with hip extension. The sitting activities
promote hip abduction (A) and knee extension (B).

68. a reciprocating gait orthosis and walker. A patient with a SCI at the level of
L1 or above has lost all functioning lower extremity muscles needed for gait.
A reciprocating gait orthosis is the best choice for this patient. Because of the
high energy expenditure, patients with injuries at this level typically depend
upon the wheelchair as their primary means of mobility. However, the
physiological and psychological benefits make it is appropriate to train this
patient in ambulation.

69. appropriate if intensities are kept below 40% maximal voluntary contraction.
Resistance training is typically initiated after patients have completed 4 to 6
weeks of supervised cardiorespiratory endurance exercise. Lower intensities are
prescribed. Careful monitoring of BP is necessary as BP will be higher and HR
lower than for aerobic exercise.

‘05 Sullivan A
72 Practice Questions

70. change the angle of pull. Changing the angle of pull will reduce the traction
pressure on the mandible and at the TMJ. The greatest traction force should be
felt on the occiput.

71. assisted breathing and coughing and pelvic floor exercises. It is important to
teach the patient to provide incision support for breathing and coughing. Pelvic
floor exercises are also important since hours of labor and pushing arc typically
present before surgery. Assisted ambulation and ankle exercises would routinely
be carried by nursing personnel and would not necessarily require a P.T. referral.

72. spondylolisthesis with possible anterior slippage of the vertebral body.


Spondylolisthesis is a bilateral fracture of the pars interarticularis with anterior
slippage. Spondylolysis is a unilateral or bilateral fracture without anterior
slippage.

73. adapt a desk and wheelchair to provide adequate sitting balance. The goal of
school physical therapy is to directly facilitate the educational process, for example,
interacting in class viewing the blackboard, etc.

74. a rolling walker to compensate for impaired balance. The patient with
Parkinson's disease typically presents with postural deficits of forward head and
trunk with hip and knee flexion contractures. Gait is narrow-based and shuffling.
A festinating gait may result from persistent forward posturing of the body near
the forward limits of stability. A rolling walker is contraindicated because it
would increase forward postural deformities and festinating gait.

75. ideomotor apraxia. With ideomotor apraxia, a patient cannot perform a task upon
command but can do the task when on their own. With ideational apraxia, a
patient cannot perform the task at all.

76. should be paid for by a third party payer. Determination about whether care is
paid for by a third party payer is determined by the payer based on contract terms
and condition. Ideally those terms include same or similar clinical factors and
rationale that are part of appropriate peer review but they need not be.

77. increased total lung capacity. An obstructive pattern on pulmonary function tests
includes increased total lung capacity caused by destruction of alveolar walls. This
same destruction causes an increased residual volume with a resulting increased
functional residual capacity and decreased vital capacity.

78. ischial tuberosity, gluteals, and lateral sides of residual limb. A quadrilateral
socket in a transfemoral amputation is designed to selectively load tissues that are

’05 Sullivan A
Practice Questions 73

pressure tolerant: the ischial tuberosity, gluteals, and lateral sides of the residual
limb.

79. immediately contact her primary physician. This patient is presenting with signs of
clinical depression. Her primary physician should be contacted immediately (especially
if suicide is mentioned).

80. mechanical stretching using traction and 5 lb. weights, 2 hours, twice daily.
Prolonged mechanical stretching involves a low-intensity force (generally 5 to 15
Ibs.) applied over a prolonged period (30 minutes to several hours). It is generally the
most effective way to manage long-standing flexion contractures. Manual passive
stretching is a short duration stretch that is not likely to be effective in this case. Lower
extremity splints and hold-relax techniques can be used to improve flexibility but are
also not likely to be effective in this case because of the short duration.

81. stretching and limiting extended spinal positions as the result of spinal stenosis.
Spinal stenosis presents with bilateral dysesthesias and pain in extended
positions and/or during walking for distances greater than 100 feet. Physical
therapy intervention should emphasize stretching of tight structures and dynamic
control of the trunk to limit long-term extended spinal positions.

82. have the teacher give a smile sticker when the child sits with head retracted for
five minutes. Positive reinforcement (use of the smile sticker) is an effective way
to shape behavior by operant conditioning. Negative behaviors are ignored
(slumping in the chair) while positive behaviors are encouraged (hold head in a
retracted position for 5 minutes.

83. ask your colleagues about their current level of knowledge using a brief
questionnaire. In order to better share the information you have learned, you
need to assess what information and skills your colleagues currently have. A brief
questionnaire can be an effective means to achieve this. Choices A, B, C,
demonstrate planning of the learning experience without benefit of a needs
assessments.

84. passive manipulation to the shoulder. All of the treatments may lie used for
RSD (Complex Regional Pain Syndrome, CRPS) except passive manipulation
which may aggravate sympathetically maintained pain.

85. lower trapezius, latissimus dorsi, and triceps. The upper quadrant muscles that
arc most important to strengthen for crutch gaits include the lower trapezius,
latissimus dorsi, and triceps. Shoulder depression and elbow extension
strength is crucial.

‘05 Sullivan A
74 Practice Questions

86. 92%. A 92% SaO2 is low but acceptable. Unacceptable oxygen saturation
rates during exercise are < 88%. Normal SaO2 is >95% - 98%.

87. ice massage B.I.D. until the pain subsides. Pain and inflammation can be
effectively treated with a cold modality. Ice massage can rapidly reduce pain and
tissue temperature in a small area without affecting the bony structure. Treatment
frequency should increase if pain is severe. Fluidotherapy three times a week is
not appropriate.

88. side-bending. The uncinate processes (joints of Luschka) are located-at the
inferior lateral aspect of the lower cervical vertebrae. Side-bending is lost with
degenerative changes at the joint the uncinate process makes with the vertebra
below. Other motion is restricted but to a lesser degree.

89. active assistive pulley exercises. The patient is most likely suffering from a
rotator cuff tear. Acute physical therapy intervention should focus on reduction
of pain and inflammation. During the early subacute phase, active assistive
pulley exercises would be indicated lo promote healing of the supraspinatus
muscle and maintain active range of motion of the glenohumeral joint.

90. interrater reliability. Interrater reliability is the degree to which two or more
independent raters can obtain the same rating for a given variable. In this case,
two therapists obtained different HIM scores for the same group of patients,
indicating a problem in interrater reliability.

91. sit the patient up, check/empty catheter, and then call for emergency medical
assistance. The patient is exhibiting autonomic dysreflexia (an emergency
situation). You should first sit the patient up and check for irritating or
precipitating stimuli (e.g., a blocked catheter). Then call for emergency medical
assistance.

92. metatarsal bar. Correction for a pes planus deformity (support of the longitudinal
arch) can be provided by a UCBL insert, scaphoid pad or Thomas heel. A
metatarsal bar is indicated to take pressure off the metatarsal heads and improve
push off.

93. custom made pressure garments. Following burns, edema and hypertrophic
scarring can be effectively controlled with elastic or pressure garments. Surgery is
an option of last resort.

’05 Sullivan A
Practice Questions 75

94. dyspnea, anxiety, or disorientation. A patient with respiratory acidosis may


present with symptoms of dyspnea, anxiety, restlessness, or headache. Significant
acidosis may lead to disorientation, stupor, or coma. Answers A, B, C are signs
and symptoms of respiratory alkalosis.

95. placing the wrists in resting splints. Modalities which use steroids are
contraindicated for pregnant women. The most effective intervention would be to
place the wrists in a neutral position in splints. The carpal tunnel is, therefore, not
compromised by poor hand positioning while at work.

96. wheelchair with a back wedge and head supports. A wheelchair wedge and head
supports holding the trunk and head in slight flexion will help decrease the
extensor tone and is the most appropriate positioning in the educational setting.
An abduction pommel controls scissoring of the legs but does not control head
and upper trunk necessary for functioning in the school environment.

97. shoulder extensors, external rotators, and anterior deltoid to position and lock
the elbow. The patient with complete C^, quadriplegia will lack triceps. He can be
taught to lock the elbow for push-up transfers by using shoulder external rotators,
extensors to position the arm; the anterior deltoid locks the elbow by reverse
actions (all of these muscles are functional).

98. demonstration, practice, and follow-up discussion. A variety of teaching


methods including demonstration, practice, and discussion has the best chance of
reinforcing the learning in a diverse group. One teaching method is not likely to
be as successful in meeting the needs of all group members.

99. have the patient wear a tight fitting mask while being treated in his room. The
therapist should wash his/her hands upon entering and leaving every patient’s
room. When the patient is suspected of having tuberculosis, the patient should be
in a private, negative pressurized room. The room is considered a potentially
infective environment and the therapist should don a tight fitting mask prior to
entering the room. The patient only needs to wear a mask if he needs to leave his
room (for a medical test, etc.). See Transmission-Based Precautions, Table 5-3.

100. patient education regarding avoidance of squatting and jumping activities as well
as initiation of iontophoresis using dexamethasone. The dysfunction observed
on the x-ray is Osgood-Schlatter's Disease. The radiograph depicts epiphysitis
of the tibia at the attachment of the patellar tendon seen in adolescents. It occurs
as the result of activities that require continued explosive contractions of the
quadriceps muscle complex during pubescent growth spurts. Patient education
should locus on controlling knee loading activities such as squatting and

‘05 Sullivan A
76 Practice Questions

jumping. Explosive contractions of the quadriceps complex should be avoided.


Ambulation and AROM activities maintain mobility while the structure heals,
Phonophoresis would be contraindicated because it may be painful to move the
sound head over the affected area. Additionally, ultrasound should not be used
over open epiphyses. Open chain knee extension exercise may aggravate
symptoms due to the increased load at the attachment of the patellar tendon to
the tibial tuberosity.

101. provide a two-wheel handcart for use in moving the boxes. Implementation of
an engineering control technique can be accomplished by designing or
modifying the workstation, work methods, and tools to eliminate/reduce
exposure to excessive exertion, awkward postures and repetitive motions.

102. subtalar pronation and medial rotation of the tibia. In order to maintain the
center of gravity over the base of support, the subtalar joint must pronate and the
entire lower quarter must medially rotate.

103. empty end-feel. An empty end-feel (no real end-feel) may be indicative of severe
pain and muscle guarding associated with pathological conditions. Springy and
firm end-feels may be expected after elbow surgery. Soft end-feel is an indication of
range limited because of tissue compression (e.g. in knee flexion there is contact
between the posterior leg and the posterior thigh).

104. behind and to the left side, one hand on the gait belt. The correct guarding
technique to protect a patient from failing is to stand slightly behind and to one
side (the involved or left side).

105. unaffected. The chronaxie is the duration of the current at twice the rheobase
(minimal intensity at a given long duration). The chronaxie of an innervated nerve is
less than then 1 ms. When a motor nerve is severed it undergoes a process called
Wallerian degeneration in which the axons distal to the lesion degenerate. The
chronaxie of the denervated nerve is greater than I ms. During the first 7 to 10
days much of the nerve constituents are still present, thus no change in the
chronaxie is observed.

106. swimming using a crawl stroke. Positioning in spinal extension increases


symptoms in patients with spinal stenosis. Activities such as swimming using a
crawl stroke place the spine in a constant extended position. All other activities
described do not require the patient to maintain an extended spinal position.

107. a shoe lift on the sound side. This patient is vaulting (rising up on the sound limb
to advance the orthotic limb forward). This gait deficit increases the energy

’05 Sullivan A
Practice Questions 77

demands of gait and is most likely contributing to his fatigue. The myalgia and
fatigue arc also direct impairments of post-polio syndrome. An appropriate
intervention plan would be to reduce the energy demands of gait. The use of a
shoe lift on the sound side would help accomplish this. A wheeled mobility device
(motorized cart) would also assist in reducing energy demands of functional
mobility. The wheelchair choices presented are poor ones.

108. stimulate the ipsilateral triceps. Stimulation of the ipsilateral triceps will reduce
the muscle tone of the biceps through reciprocal inhibition.

109. ankle-foot orthosis. Individuals with an L5 lesion need an orthosis to correct foot
alignment and for push-off. There is partial innervation of the hamstrings and
gluteus minimus and medius, posterior tibialis and peroneus tertius muscles. The
other orthoses would he indicated for higher level lesions.

110. closed-chain partial weight-bearing lower extremity exercises for slipped capital
femoral epiphysis. Slipped capital femoral epiphysis is characterized by a glutens
medius gait. Closed-chain exercises with weight-bearing to tolerance will help
regain or maintain functional muscular strength and normal motion.

111. null hypothesis. The null hypothesis is a statistical hypothesis that states that
there is no relationship (or difference) between variables. Any relationship found
will be a chance relationship, not a true one.

112. that the therapist was functioning outside the common protocols of the hospital,
and therefore did not support the actions of the physical therapist. The physical
therapist was acting outside of her area of responsibility and did something that
caused the patient harm.

113. patch one eye. Double vision (diplopia) can be managed by patching of one eye.
Patients are typically on an eye patching schedule which alternates the eye that is
patched. Loss of depth perception can be expected with eye patching but is not as
disabling as diplopia.

114. sidelying on the sound side with the affected upper extremity supported on a
pillow with the shoulder protracted and elbow extended. Most patients with
stroke recover from the flaccid stage and develop spasticity. Positioning for the
patient with early stroke stresses (I) protection against ligamentous strain and the
development of a painful subluxed shoulder and (2) positions counter to the typical
spastic posture of flexion and adduction with pronation. Sidelying with the affected
upper extremity supported on a pillow with the shoulder protracted and elbow
extended accomplishes both of these goals. The other positions do not.

‘05 Sullivan A
78 Practice Questions

115. chronic venous insufficiency. Venous ulcers are often painless, or present with
minimal pain when compared to arterial ulcers which are painful (claudication and
rest pain). Chronic venous insufficiency is also characterized by thickening,
coarsening, and brownish pigmentation of the skin around the ankles. The skin is
usually thin, shiny, and cyanotic. DVT may be asymptomatic initially. When
symptoms occur, patients typically report a dull ache, tightness, or pain in the
calf.

116. large electrodes, widely spaced; 10:30. Large electrodes are used on large
muscles in order to disperse the current (minimize current density under the
electrode) enabling a more comfortable delivery of current. Widely spaced
electrodes permit the current to travel deeper into the muscle to stimulate a
greater number of deeper muscle fibers. A duty cycle of 1:3 will allow a
sufficient rest period between contractions to minimize the fatiguing effect of
repetitive muscle contractions.

117. call the physician immediately and report your findings. A mole with irregular
edges and multiple colors (black or blue) is characteristic of a melanoma. It is
associated with sun-damaged skin but can appear anywhere on the body. Early
recognition before tumor invasion and metastatic spread is critical in improving
survival rates. The physician should be informed immediately. Medical
management consists of biopsy and surgical removal.

118. decreasing consciousness with slowing of pulse and Cheyne-Stokes respirations.


Signs of increased intracranial pressure secondary to cerebral edema and brain
herniation include decreasing consciousness with slowing of pulse and Cheyne-
Stokes respirations. Cranial nerve dysfunction is typically noted in C.N. II
(papilledema) and C.N.III (dilation of pupils). Choices A and C are signs of
meningeal irritation and CNS infection.

119. nerve dysfunction will be rapidly reversed, generally in 2-3 weeks. Neurapraxia
is a mild peripheral nerve injury (conduction block ischemia) that causes
transient loss of function. Nerve dysfunction is rapidly reversed, generally
within 2 - 3 weeks. An example is a compression injury to the radial nerve from
falling asleep with the arm over the back of a chair (Saturday night palsy).

120. occasional feedback given when consistent errors appear. In learning a


psychomotor skill, the patient needs to be able to actively process information and
self-correct responses. Occasional feedback provides the best means of allowing for
introspection and is appropriate for later in practice (associated and autonomous
phases of motor learning).

’05 Sullivan A
Practice Questions 79

121. he may be presenting with early signs of myocardial infarction. An elderly


patient with a cardiac history may present with initial symptoms of mental
confusion, the result of oxygen deprivation to the brain. The SOB and generalized
weakness may also be due to generalized circulatory insufficiencies coexisting
with the developing myocardial infarction.

122. modalities to reduce inflammation, active assistive range of motion exercises


using pulleys, and postural realignment. Initial physical therapy intervention of
tendinitis with possible impingement should emphasize active assistive exercises,
modalities to reduce inflammation, and postural realignment to reduce
impingement of the rotator cuff at the acromion.

123. 10 to 15 seconds. The recommended time duration for endotracheal suctioning is


10 to 15 seconds. Any longer risks serious any shorter and the risk is ineffective
secretion removal.

124. occlusive dressings. Autolytic wound debridement allows the body's natural
enzymes to promote healing by trapping them under a synthetic, occlusive
dressing. The dressings arc applied for short durations (less than 2 weeks ) and arc
contraindicated in infection. The other answer are appropriate wound
management techniques; however, they are not autolytic.

125. switch to modulation mode TENS. Because of the long-term continuous use of
TENS, the sensory receptors accommodated to the continuous current and no
longer responded to the stimuli. Changing to modulation mode (i.e. burst
modulation), which periodically interrupts the current flow docs riot allow
accommodation to occur.

126. gastrocnemius-soleus. The muscles of the foot and ankle move the long lever of
the body forward and backward using ankle synergies. The gastrocnemius-soleus
moves the body backward while the anterior tibialis moves the body forward.
Action of the hip extensors would result in a backward lean with the center of
motion occurring at the hip.

127. false negative. Sensitivity refers to the ability of a test to correctly identify
individuals who truly have a disease or condition (a true positive). In this example,
the individual was found not to have the Alzheimer's gene, when years later he
tested positive for the gene and the disease. The original test produced a false
negative result.

128. history of sudden onset of new cognitive problems and patchy distribution of
deficits. Multi-infarct dementia differs from primary degenerative dementia,

‘05 Sullivan A
80 Practice Questions

Alzheimer's type, in (1) onset: sudden rather than slowly progressive


(characteristic of small focal infarcts) and (2) the nature of symptoms: areas of
deficits coexist with areas of intact cerebral function. Agitation and sundowning
(late afternoon wandering) are characteristics of Alzheimer's disease.

129. vastus medialis muscle strengthening. Q angles greater than 15 degrees could be
indicative of abnormal lateral patellar tracking. Vastus medialis muscle
strengthening can reduce the tendency for the patella to track laterally. Vastus
lateralis (VL) strengthening can promote greater lateral patellar tracking and further
irritation of the patellofemoral joint. VL strengthening may promote an outward
pull or dislocation of the patella. Hamstring strengthening does not directly affect
tracking of the patella.

130. standing, picking the foot up behind and slowly lowering it. Stage 4 recovery is
characterized by some movement combinations that do not follow that paths of
either flexion or extension synergies. Knee flexion in standing is an out-of-synergy
movement. All other choices represent synergistic movements: choices A and B
are flexion synergy movements while choice C focuses on knee extensor
movement within an extended position.

131. examine the patient, document and discuss your findings with the doctor. The
physical therapist should complete the examination of the patient, adequately
document the findings, and determine the physical therapy diagnosis. While many
states have direct access laws which permit physical therapy intervention without
referral, most insurance companies including Medicare (affecting the patient in
this example) require a physician referral in order for services to be reimbursed.
Thus the therapist needs to consult with the physician to get a referral before
initiating any intervention for this problem. This patient demonstrates stress
incontinence, a problem that could be successfully treated with physical therapy
(e.g., Kegel exercises and other interventions).

132. anterior spinothalamic tract. Sensations interpreted as dull, aching pain travel in
the anterior (paleo) spinothalamic tract. Discriminative fast pain is carried in I he
lateral (neo) spinothalamic tract. Discriminative touch is carried in the
proprioceptive pathways (fasciculus gracilis/cuneatus, medial lemniscus).

133. increase the patient’s maximal oxygen consumption by discharge. All of the
choices are important goals for Phase I, acute or inpatient cardiac rehabilitation
except for altering maximal oxygen consumption. Patients generally are expected
to reach a functional capacity of 3-5 METs by discharge from a Phase I program.

’05 Sullivan A
Practice Questions 81

134. placement of the drive wheels 2 inches posterior to the vertical back supports.
Placement of the drive wheels 2 inches posterior to the vertical back supports is
an appropriate modification for a patient, with bilateral transfemoral amputations.
This increases the length of the base of support and provides increased posterior
stability. Lowering the seat height by 3 inches is an appropriate modification for a
patient with stroke who will use his sound limbs for wheelchair propulsion.
Increasing the seat depth is not an appropriate modification.

135. P=0.05. A preselected probability level of 0.05 indicates that the results (in this
example differences in measurements) would be the result of chance only 5 times
out of every 100 studies. This level of confidence helps us reject the null
hypothesis (there is no difference in goniometric measurement techniques) and is
common in most experimental studies (0.01 is the other, more stringent level of
significance commonly applied).

136. instruction in proper postural alignment. Without regaining normal postural


alignment and scapular-humeral rhythm, the patient will continue to impinge the
supraspinatus and/or biceps tendon at the acromion and never regain normal
function of the shoulder. It is unlikely that all pain would be controlled.

137. pain and stiffness are worse in the early morning and should decrease with
moderate activity. Degenerative joint disease (osteoarthritis) is a noninflammatory
progressive disorder affecting primarily the carpometacarpal, knee, and hip joints.
Stiffness is common following inactivity and is relieved with movement. Exercise
can be helpful in reducing pain and immobility and is not contraindicated. Joint
projection strategies are effective intervention for maintaining function.

138. symmetrical distribution of weakness, ascending with possible involvement of


lower cranial nerves. Guillain-Barre syndrome is an acute polyneuritis
characterized by rapid development of progressive muscle weakness. The
weakness is typically symmetrical and ascends the body (starting first in the lower
extremities, progressing to trunk, upper extremities, and finally cranial nerves).
Stocking and glove sensory loss could also be found but not with minor loss of
motor function.

139. monophasic current with the anode placed on the tendon. The anode (positive)
electrode would effectively repel the positively charged ion into the tendon.
Because of its unidirectional flow, monophasic currents are best suited for
iontophoresis.

140. have the patient do sitting push-ups at least every 10 minutes. Excessive ischial
pressure and redness from prolonged sitting requires an aggressive approach.

‘05 Sullivan A
82 Practice Questions

Arm push-ups, at least every 10 minutes, is indicated if redness is present. A


high-density gel cushion might help as would a tilt-in-space chair but neither of
these would address the immediate problems.

141. medial rotation. Common abnormal postural findings consistent with anterior
knee pain in an adolescent female include pes planus, lateral tibial torsion and genu
valgum with compensation at the femur of excessive medial rotation. Lateral
femoral rotation is commonly observed with genu varum. Retroversion of the hip is
an abnormally small angle between the femoral neck and condyles and is not
affected by posture.

142. you will meet with the occupational therapist to discuss the exercise approaches
and you will let them know the outcome of the meeting as soon as possible. Any
discrepancies in educational approaches used with the patient's family should be
handled by the learn members, not the family or rehabilitation supervisors. A team
meeting is a good place to iron out approaches to patient care and ensure
consistency.

143. a videotape of another child with cerebral palsy on a Swiss ball. A videotape of
another child with CP on the ball represents the best choice to engage this child:
The other choices, while important educational media, are not likely to adequately
present the three-dimensional qualities of performance needed for exercising on
the Swiss ball.

144. summon emergency medical services. If the victim is unresponsive, Emergency


Medical Services must be activated immediately by calling 911 or calling a
"code" if in a health care institution. If the victim is a young child or infant, try
one minute of rescue breathing before summoning help.

145. call the girl’s pediatrician immediately. These signs could be the result of
increased cerebral edema due to a clogged or infected shunt. Medical attention
should he obtained immediately to avoid damage to the brain.

146. protection and splinting of the abdominal musculature. Diastasis recti abdominis
is a condition in which there is a lateral separation or split of the rectus abdominis.
It is important to teach protection (splinting) of the abdominal musculature.
Patients should be instructed to avoid full sit-ups or bilateral straight leg raising.
Pelvic floor exercises should be done but are not remediation for diastasis
recti.

147. supine, head of bed flat. A patient with a C5 spinal cord injury will not have the
abdominal musculature necessary to return the diaphragm to a high domed

’05 Sullivan A
Practice Questions 83

position during exhalation. Inspiration will be affected by the change in the


diaphragm's resting position. In supine, gravity will take the place of abdominals,
holding the abdominal contents under the diaphragm, improving the zone of
apposition, the height of the diaphragm dome and therefore, the ability to
ventilate. The other positions listed negate the positive effects of gravity on the
abdomen.

148. schedule a conference with the doctor and family about her condition and your
discussions with the patient. The most appropriate strategy is to hold a conference
with the doctor and family about her condition and your discussions with the
patient. Everyone interacting with this patient should be answering her questions
in the same way. A direct and honest approach is best but must be consistent with
the parent’s wishes since she is a minor child.

149. overwork damage in weakened, denervated muscle. Amyotrophic lateral


sclerosis is a progressive degenerative disease that affects both upper and lower
motor neurons. An important early goal of physical therapy is to maintain the
patient's level of conditioning while preventing overwork damage in denervated
muscle (lower motor neuron injury). Myalgia is common in I,MN lesions. It can
be ameliorated " but not prevented. Ataxia and radicular pain are not associated
with ALS.

150. slight wrist extension with fingers supported and thumb in partial opposition and
abduction. A resting splint that positions the wrist and hand in a functional
position includes 10-20 degrees of wrist extension, fingers supported, and thumb
in partial opposition and abduction.

151. complaints of fatigue with increasing dyspnea and cough. Left heart failure is a
condition in which blood is not adequately pumped out of the heart by the left
ventricle. The blood; therefore, backs up creating pulmonary signs and symptoms
(increased pulmonary artery pressures, cough, dyspnea, orthopnea), weakness,
and fatigue. All other choices are signs associated with right-sided heart failure.

152. take a rest period and only use the device 10 times per hour. If a patient feels
lightheaded with an incentive spirometer, it may be that they are blowing off too much
CO2 by hyperventilating. An incentive spirometer should always be used in the most
upright position possible to attain the highest values possible.

153. 30 minutes each day. Motor learning is enhanced by daily practice of moderate
duration.

‘05 Sullivan A
84 Practice Questions

154. compression of the long thoracic nerve. Vigorous upper limb activities can cause
inflammation of soft tissues surrounding the shoulder resulting in compression of the
long thoracic nerve and weakness of the serratus anterior. The serratus anterior
stabilizes the scapula with greater than 90 degrees of abduction. Supraspinatus
tendinitis or weakness does not result in scapular winging. The supraspinatus muscle
in concert with the deltoids initiates abduction in the upper extremity. Subdeltoid
bursitis causes pain with all active range of motion arid does not result in scapular
winging.

155. sacral sitting. Spasticity is typically strong in antigravity muscles. In the lower
extremities this is usually the hip and knee extensors, adductors, and plantar flexors.
Strong extensor tone results in sacral sitting with the pelvis tilted posteriorly.

156. peripheral edema. Peripheral edema is a symptom of overdosage/toxicity for


patients on Premarin. The risk of osteoporosis and fracture is reduced for individuals
on estrogen replacement therapy. Control of hot flashes is a major indication for its
use.

157. jogging is safe but you might want to switch to swimming during later months.
According to the American College of Sports Medicine, women can continue to
exercise regularly (3 times a week) throughout pregnancy if no additional risk
factors are present. After the first trimester women should avoid exercise in the
supine position since this position is associated with decreased cardiac output.
Non-weightbearing exercise (swimming) is an appropriate alternative to decrease
the risk of injury and facilitate continuation of exercise throughout pregnancy.

158. concentric exercises, 3 sets of 10, with gradually increasing intensity. This
patient is experiencing delayed-onset muscle soreness (DOMS) as a result of
vigorous exercise or muscular overexertion. It typically begins 12 to 24 hours
after exercise, peaks in 24 to 48 hours, and can last up to 5 to 7 days. DOMS
is usually greater after muscle lengthening or eccentric exercise and can be
lessened by gradually increasing intensity and duration of exercise.

159. an iliotibial band friction disorder. A positive Noble compression test is an


indication of an iliotibial band friction disorder. A FABERE test can distinguish
between hi p joint pathology and sacroiliac joint dysfunction.

160. problems with motor planning and scaling of movements. The basal ganglia
functions to convert general motor activity into specific, goal-directed action
plans. Dysfunction results in problems with motor planning and scaling of
movements and postures (e.g., bradykinesia).

’05 Sullivan A
Practice Questions 85

161. vagus nerve. These are the tests for the vagus nerve C.N. X. See chapter 2 for
cranial nerve tests.

162. increased volume of air at resting end expiratory pressure (REEP). The gravity
independent area of the lung in the upright sitting position refers to the apices of I
he lungs which house the most air at REEP. That area has the least perfusion
because of the effects of gravity on blood flow. The apices also have the smallest
change in ventilation during the respiratory cycle because they are the most full at
rest. Finally, this area of the lung has the highest oxygenation and lowest carbon
dioxide content due to the small volume of blood that needs to be diffused.

163. just anterior and superior to the greater trochanter. The orthotic hip joint should
coincide with a point just anterior and superior to the greater trochanter (the
anatomic center of the hip joint).

164. the PTA who is completely liable because the plan of care was altered without
communicating with the supervising PT. Only the supervising PT may alter the
established plan of care. In this case the treatment plan stipulated progressive
ambulation on level surfaces only. Therefore, the PT is not responsible, the PTA
is responsible and negligent.

165. gentle AROM exercises in weight bearing. Aggressive soft tissue stretching and
strengthening can promote myositis ossificans. Gentle weight bearing AROM
exercises to patient's tolerance will minimize the chance of myositis ossificans
and promote improved function.

166. a tone inhibiting ankle-foot orthosis (AFO). A tone inhibiting AFO will make
ambulation easier and will help maintain ROM of the affected ankle. The most
appropriate assistive device would be a cane (i.e. straight or SBQC), not a
walker.

167. inadequate contraction of the ankle dorsiflexors. Weakness or delayed


contraction of the ankle dorsiflexors or spasticity in the ankle plantarflexors may
cause foot drop during midswing. Excessive extensor synergy would cause
plantarflexion during stance.

168. tell her that she must have the permission of her father before she can look at the
chart. The only time confidentiality can be breached is if the patient agrees or he is
in danger. Even the family doesn't have the right to the medical information without
the patient's permission unless the patient is a minor child or is incompetent to make
decisions.

‘05 Sullivan A
86 Practice Questions

169. decrease edema. The reduction of edema is through the diffusion of synovial fluid. In
addition, increased venous flow and increased nutrition is occurring in and around the
joint enhancing the healing process. The reduction of pain is a secondary
consequence of decreased inflammation. Range of motion is usually limited during
the first few hours of treatment and no tension is to be placed on the surrounding
tissues.

170. five times per week. The patient with acute tendinitis receiving iontophoresis should
come daily the first two weeks to enable optimal amounts of medication to affect the
inflamed tissue. The patient needs to return to work as soon as possible.

171. hip flexion contracture. A hyperextended knee can be caused by extensor


spasticity, quadriceps weakness (a compensatory locking of the knee), or by
plantarflexion contractures or deformity.

172. call emergency medical technicians as it may be a pneumothorax. The combined


signs and symptoms of absent breath sounds, sudden onset of chest pain and
shortness of breath indicate a pneumothorax, especially in a 15 year-old male
(growth spurt) with pathological changes of king tissue. This should be considered
an emergency situation.

173. supervision of home health aides for completion of a daily home exercise
program. Medicare funds home health care under Part A. Teaching of home care
aides to ensure completion of a daily home exercise program is a reimbursable
service while routine provision of an established maintenance program in which
the therapist performs the exercises would not be covered.

174. PNF bilateral symmetrical upper extremity D2 flexion patterns, rhythmic


initiation. The patient with Parkinson's disease typically develops elbow flexion,
shoulder adduction contractures of the upper extremities along with a flexed,
stooped posture. Bilateral symmetrical upper extremity PNF D2F patterns
encourage shoulder flexion and abduction with elbow extension, and upper trunk
extension (all needed motions). Both quadruped and modified plantigrade
positions encourage postural flexion.

175. perform the ROM exercises yourself. The practice of using supportive personnel
falls under the Guide for Professional Conduct (Appendix B0 and under the
individual practice acts of the states. Delegated responsibilities should be
commensurate with the qualifications (experience, education, training) of the
individual to whom responsibilities are being assigned. In this case, it is not
reasonable to assume an aide newly arrived to the P.T. department has the
knowledge or skills to do this treatment. A brief orientation to ROM exercises is

’05 Sullivan A
Practice Questions 87

not adequate to ensure proper treatment. The therapist should do the ROM
exercises herself. In some states, any treatment by an aide would be inappropriate.

176. wheelchair with head and trunk support with abductor wedge. The wheelchair
would be the most functional adaptive equipment with postural support and
abductor wedge to help decrease extensor spasms of lower extremities. This
individual is not stable enough to use a rollator walker or scooter board.

177. back extension, costal expansion exercises, and maintenance of proper posture to
prevent deformity. Postural reeducation will help to prevent further increase in
thoracic kyphosis and costal expansion exercise will improve breathing
efficiency. Abdominal strengthening and stretching of scapular stabilizers are not
indicated for ankylosing spondylitis.

178. contralateral hemiparesis and sensory deficits with greater involvement in the
arm than the leg. A CVA affecting the middle cerebral artery will result in
symptoms of contralateral hemiparesis and hemi sensory deficits with greater
involvement of the arm than the leg. Choice B is incorrect since involvement
would be the left hemisphere with right hand dominance. Choice C (greater leg
involvement than arm) is characteristic of a CVA affecting the anterior cerebral
artery.

179. elevated levels of low-density lipoprotein or LDL. Increased blood cholesterol


levels with high levels of low-density lipoproteins (LDLs) increases the risk of
coronary artery disease (CAD); conversely low concentrations of high-density
lipoproteins (HDLs) is also harmful. The link between CAD and triglycerides
is not as clear.

180. thank him very much, but refuse his invitation while he is receiving treatment.
Physical therapy departments should have a policy stating it would be unethical to
have sexual contact or dating between staff and patients. The Guide for
Professional Conduct of the American Physical Therapy Association (Appendix
B, Administration Chapter) states in Principal 2 that physical therapists shall not
engage in any sexual relationship or activity, whether consensual or non-
consensual with any patient while a physical therapist-patient relationship exists.
Some authorities suggest a waiting period of 6 months after discharge from
treatment before commencing an intimate relationship.

181. referral to a physician to evaluate anterior compartment pressures during


activity. Deep anterior leg cramping after running distances- one mile or greater
would most likely be due to a local ischemic condition of the anterior
compartment as a result of excessive pressure. Shin splints and tendinitis symptoms

‘05 Sullivan A
88 Practice Questions

would occur earlier in running and be felt as pain. Referral to a physician is


indicated if nontraumatic compartment syndrome is suspected. Traumatic
anterior compartment syndrome requires immediate attention by a physician.
Spinal stenosis usually manifests itself as posterior leg cramping.

182. teach her activity pacing and energy conservation techniques. You should teach
this patient activity pacing and energy conservation techniques. It is important she
is able to balance rest with activity in order to not further weaken muscles
affected by progressive post polio muscular atrophy.

183. ligamentum flavum. The ligamentum flavum becomes hypertrophied with lumbar
spondylosis and may invade the intervertebral foramen compressing the left L5
spinal nerve root.

184. onset of moderate to severe angina. According to the American College of


Sports Medicine, an absolute indication for terminating an exercise bout is onset
of moderate to severe angina. Other absolute indications include: acute MI, a drop
in SBP with increasing workload, serious arrhythmias (2nd or 3rd degree heart
blocks, sustained ventricular tachycardia or PVCs, atrial fibrillation with fast
ventricular response), unusual or severe shortness of breath, CNS symptoms
(ataxia, vertigo, confusion), or patient's request. The other choices are
considered relative indications and require close monitoring.

185. notify the physician immediately, you suspect heterotopic ossifications may be
developing. Early signs of heterotopic bone formation include swelling, pain,
erythema occurring near a large joint. The physician should be notified
immediately. Vigorous stretching is contraindicated.

186. clearance between the floor and the foot plate of at least 2 inches. The correct
measure for seat height in a wheelchair is 2 inches clearance between the floor
and the foot plate, measured from the lowest point on the bottom of the footplate.

187. recreational activities such as swimming or biking. Recreational exercise will be


both fun and helpful in maintaining functional level as long as possible. At 6
years of age, wheelchair confinement is not usual.

188. currently there is no limit to her Medicare coverage for outpatient PT services.
From September J, 2003 through December 7, 2003 there was a limit on the
amount Medicare would pay for outpatient physical therapy services. The limit of
$1590.00 was in effect for outpatient PT services given at a private clinic but not
for hospital outpatient department services. Effective December 8, 2003 the limit
was removed by the recent Medicare Prescript ion Drug and Modernization Act of

’05 Sullivan A
Practice Questions 89

2003. Currently there is no limit to the amount of medically necessary outpatient


PT, OT, or SLP services.

189. use an alternate eyelid taping schedule which will allow her to use eye
movements to communicate. Pontine lesions that result in locked-in syndrome
leave the patient with an inability to move or speak but with full cognitive function.
By taping one eye open, the patient's ability to receive sensory inputs is increased
(sensory deprivation is lessened) and eye movements can be used for
communication. They should not expect any active responses from the patient.

190. maintain the keyboard in a position allowing a neutral wrist position. Work
involving increased wrist deviation from a neutral posture in either
flexion/extension or radial/ulnar deviation have been associated with increased
reports of carpal tunnel syndrome and other wrist and hand problems.

191. depressed ST segment with a flat T wave and shortened QT interval. Digitalis
produces characteristic changes on the ECG: gradual downward sloping of ST
segment with a flat T wave and shortened QT interval.

192. intermittent pneumatic compression, extremity elevation, and massage.


Lymphedema following surgery and radiation can be effectively managed by
external compression and extremity elevation. Manual lymph drainage (massage
and PROM) are also appropriate interventions. Exercise and positioning alone
would not provide the needed lymph drainage; isometric exercise is
contraindicated.

193. suggest that the patient speak with his surgeon. It is within the physician's scope
of practice to discuss the indications and problems that could arise from this
surgical procedure. It is not within the physical therapist's scope of practice to be the
expert who discusses problems associated with surgery.

194. gravity assisted right upper extremity exercises to promote scapular control
following damage to the long thoracic nerve. With axillary dissection the long
thoracic nerve may be damaged. This leads to serratus anterior weakness and
loss of scapular control. Gravity assisted exercises to promote scapular control
should be emphasized early in rehabilitation to help restore proper scapular
humeral rhythm.

195. amount of productive secretions decreases. The purpose of postural drainage is


to help remove secretions. If the amount diminishes this might be an indicator
that the treatment has been successful and that the frequency of treatment can be
reduced. The other choices of lever, sputum consistency and pain do not provide
a rationale to decrease treatment frequency.
‘05 Sullivan A
90 Practice Questions

196. chronic, episodic bouts of dysfunction. Meniere's disease is a recurrent and


usually progressive vestibular disease characterized by episodic bouts of
dysfunction. As w i t h most vestibular disorders, varying degrees of vertigo,
nausea, and postural i n st ab i l i t y will occur.

197. Workers’ Compensation. The employee was injured on the job even though she
arrived early. She is therefore covered by Worker's Compensation for her injury.

198. postural reeducation to prevent positions that increase symptoms. Patients


should be educated to prevent extended positions for long periods. Symptoms
resulting from Paget's disease are aggravated by positions where the lumbar spine
is in extension. Because this is a chronic condition, modalities are not the most
effective management strategy. Lumbar extension exercises decrease the space
within the vertebral foramen thereby increasing symptoms associated with
stenosis and Paget's disease.

199. the posterior stop setting the foot in too much plantarflexion. An ankle-foot
orthosis can be used to assist in knee control for the patient with hemiplegia. A
foot set in slight plantarflexion (generally about 5 degrees) helps to create an
extension moment at the knee and can thus assist weak knee extensors in
extending the knee. However if the ankle is set in too much plantarflexion, it can
cause genu recurvatum.

200. daily walking through pain in order to attain predetermined distances. Patients
with peripheral vascular disease with intermittent claudication can benefit from a
walking program to improve peripheral blood flow and functional capacity. The
program should be of moderate intensity and duration. Patients should be
encouraged to exercise to the point of pain, not through or beyond.

’05 Sullivan A

Вам также может понравиться